You are on page 1of 110

540 MCQS

Grand test one paper Gulberg


1. The square root of the cube of this number is the cube of its square root. It is not 1 and it is less than
6. What is it?
(a) 2 (b) 3 (c) 4 (d) 5
2. In the sequence 462, 420, 380, X, 306.Then “X” stands for?
(a) 352 (b) 342 (c) 332 (d) 322
3. Ten percent of twenty plus twenty percent of ten equals:
(a) 10 per cent of 20 (b) 15 per cent of 20 (c) 1 per cent of 200 (d) 2 per cent of 200
4. A, B and C can do a work in 2 days. B can do it in 6 days and C can do it in 5 days. How long
would it take A to do the work?

(a) days (b) days (c) days (d) days


5. If 70 men can build 4 houses in 12 months, how many men would be required to build 6 houses in
4 months?
(a) 345 men (b) 315 men (c) .301 men (d) 360 men
6. If 15 men can do a piece of work in 8 days, how many men will finish it in 10 days?
(a) 14 men (b) 12 men (c) 16 men (d) 8 men
7. If 3 men and 6 boys can do a work in 20 days, then 6 men and 8 boys shall take:
(a) 6 days (b) 8 days (c) 9 days (d) 20 days
8. Pipe "A" can fill a tank in 5 minutes whereas "B" can fill it in 6 minutes. How long it would take
both pipes, working together, to fill it?

(a) minutes (b) minutes (c) minutes (d) minutes


9. 28, 33, 31, 36,?, 39
(a) 28 (b) 34 (c) 40 (d) 44
10. Ali spends Rs.500 to get a good shirt and sells it at Rs.800. Find the % age of profit?
(a) 59% (b) 60% (c) 63% (d) 40%
11. Kiran spends Rs.33, 000 to get a new computer. After few days, the price of computer rises and she
sells it for Rs.35, 000. How much profit, did she get in %age?
(a) 82% (b) 90% (c) 46% (d) 6.06%
12. Find the average of 8, 16, 32, 49, 51?
(a) 48.9 (b) 31.2 (c) 62.4 (d) 71.06
13. Find the average of 110, 216, 419, 810?
(a) 419.76 (b) 310.61 (c) 388.75 (d) 840.68
14. If x - 4=11, then what is the value of x - 8?
(a) 3 (b) 5 (c) 9 (d) 7
15. If 20 men can do a piece of work in 8 days, how many men will finish it in 10 days?
(a) 4 men (b) 16 men (c) 8 men (d) 18 men
16. A pump can fill a tank with water in 2 hours. Because of a leak, it took 2+2/3 hours to fill the tank.
The leak can drain all the water of the tank in:
(a) 4-1/3 hours (b) 7 hours (c) 8 hours (d) 14 hours
17. Pipes A and B can fill a tank in 8 and 24 hours respectively. Pipe C can empty it in 12 hours. If all
the three pipes are opened together, then the tank will be filled in:
(a) 6 hours (b) 12 hours (c) 18 hours (d) 24 hours
18. Saima scored 178 marks out of 300 marks in a certain text. What percentage of marks did she
score?
(a) 48% (b) 52.41% (c) 61.41 % (d) 59.33%
19. Ali gained 510 marks in matriculation examination out of 850 marks. What percentage of marks
did he gain?
(a) 60% (b) 57% (c) 71 % (d) 73%
20. In the sequence of numbers 5, 8, 13, X, 34, 55, 89 the value of 'X' is:
(a) 20 (b) 21 (c) 23 (d) 29
21. On another planet, the local terminology for earth, water, light, air and sky, are 'sky', 'light', 'air',
'water' and 'earth' respectively. If someone is thirsty there, what would he drink?
(a) Sky (b) Water (c) Air (d) Light
22. A man purchases two clocks A and B at a total cost of Rs. 650. He sells A with 20% profit and B at
a loss of 25% and gets the same selling price for both clocks. What are the purchasing prices of A
and B respectively?
Sol: TO GET BACK THE TOTAL PRICE OF (A+B)
LET THE PURCHASE PRICE OF ‘A’ BE — — — — — X

LET THE PURCHASE PRICE OF ‘B’ BE — — — — — Y

THEN X+Y = 650.

ON SELLING 1.2X + 0.75Y = 650

SOLVING X = 361.111, Y = 288.889

2. TO HAVE THE SAME SELLING PRICE FOR EITHER

1.2X = 0.75Y, X+Y = 650. HENCE X = 250, Y = 400

(a) Rs. 225; Rs. 425 (b) Rs. 250; Rs, 400 (c) Rs. 275; Rs. 375 (d) Rs. 300; Rs. 350
23. If 15 pumps of equal capacity can fill a tank in 7 days, then how many extra pumps will be
required to fill the tank in 5 days?
(a) 14 (b) 7 (c) 6 (d) 21
24. Which one of the following satisfies the relationship DDA: aDD::Rrb:?
(a) DDA (b) RRR (c) BBR (d) BRR
25. A, B, C, D, E and F, not necessarily in that order are sitting on six chairs regularly placed around
a round table. It is observed that A is between D and F, C is opposite D, and D and E are not on
neighbouring chairs. Which one of the following pairs must be sitting on neighbouring chairs?
(a) A and B (b) C and E (c) B and F (d) A and C
26. If in a certain code SAND is VDOG and BIRD is ELUG, then what is the code for LOVE?
(a) POYO (b) ORTG (c) NPUH (d) ORYH
27. In a family, a couple has a son and a daughter. The age of the father is three times that of his
daughter and the age of the son is half of his mother. The wife is nine years younger to her
husband and the brother is seven years older than his sister. What is the age of the mother?
(a) 40 years (b) 45 years (c) 50 years (d) 60 years
28. An accurate clock shows the time as 3.00. After hour hand has moved 135°, the time would be:
(a) 7.30 (b) 6.30 (c) 8.00 (d) 9.30

29. If the cost of a loaf is Rs. 5.50, what is the cost of loaves of bread?
(a) 471.68 Rs (b) 529.76 Rs (c) 13.75 Rs (d) 15.0 Rs
30. The whole journey in a walking competition is 15km. Ali has completed 9km. What percentage of
the whole journey has he completed?
(a) 60% (b) 70% (c) 75% (d) 64%
31. If ten oranges cost Rs. 75 how much will one dozen oranges cost?
(a) 85 (b) 80 (c) 90 (d) 95
32. 12 + 31 + 19 + 22 + 87 =
(a) 159 (b) 149 (c) 171 (d) 181
33. 121 ÷ 1.5
(a) 90.2 (b) 80.6 (c) 78.4 (d) 92.8
34. The teacher bought 10 rulers for Rs. 25:
(i) What was the price of each ruler?
(ii) What will he have to pay for 32 rulers?
(a) a = 2.50 Rs. b = 80 Rs (b) a = 2.50 Rs. b = 70 Rs
(c) a = 1.50 Rs. b = 60 Rs (d) No one is correct
35. It takes 14 taps to fill a swimming pool in 10 hours. How long will it take 5 taps to fill the pool?
(a) 21 (b) 22 (c) 28 (d) 34
36. A bridge can be constructed by 130 men in 40 days. How long will it take 110 men to do the work?
(a) 47.27 (b) 41.22 (c) 52.34 (d) 62.21
37. A can do a piece of work in 10 days and B can do it in 15 days. The number of days required by
them to finish it, working together is:
(a) 8 (b) 7 (c) 6 (d) 4
38. If 30 % of a number is 12.6, then the number is:
(a) 41 (b) 51 (c) 52 (d) 42
39. An accurate clock shows 8 O'clock in the morning. Through how many degrees will the hour hand
rotate when the clock shows 2 O'clock in the afternoon?
(a) 150° (b) 180° (c) 168° (d) 144°
40. The monthly incomes of Kamal and Arshad are in the ratio of 4:3. Their monthly expenses are in
the ratio of 3:2. However, both save Rs. 600 per month. What is their total monthly income?
Sol… Let their income be 4x and 3x
let their expenditure be 3y and 2y
income - exp = saving
so, 4x - 3y = 600
3x - 2y = 600
on solving
8x - 6y = 1200
- 9x + 6y = 1800
x = 600
y = 600
income of a = 4×600= 2400
income of b = 3×600 = 1800
(a) Rs. 4,200 (b) Rs. 5,600 (c) Rs. 8,400 (d) Rs. 2,800
3 2
41. If X = -2, then X - X - X - 1 is equal to
(a) 1 (b) -3 (c) -11 (d) -15
42. Examine the following relationships among members of a family of six persons A, B, C, D, E and
F.
I. The number of males equals that of females.
II. A and E are sons of F.
III. D is the mother of two, one boy and one girl.
IV. B is the son of A.
I. There is only one married couple in the family at present.
Which one of the following inferences can be drawn from the above?
(a) A, B and C are all females (b) A is the husband of D
(c) E and F are children of D (d) D is the granddaughter of F
43. What percentage of 84cm is 15cm?
(a) 21 % (b) 32% (c) 44% (d) 17.86%
44. The price of a TV set increases from Rs.4500 to Rs.8000, what is the percentage increase?
(a) 77.78% (b) 90% (c) 40% (d) 14%
45. (30% of 80)/? =24
(a) 3/10 (b) 3/17 (c) 1 (d) 2
46. 45% of ? + 30% of 90 = 30% of 210.
(a) 120 (b) 80 (c) 60 (d) 90
47. If 37% of a number is 990.86, what will be approximately 19% of that number?
(a) 600 (b) 400 (c) 500 (d) 700
48. 35% of 30=25% of ? +1
(a) 28 (b) 38 (c) 42 (d) 32
49. If 8% of x=4% of y, then 20% of x is:
(a) 10% of y (b) 16% of y (c) 80% of y (d) None of these
50. An estimated 3 out of every 25 men are left-handed. What percent of men are left-handed?
(a) 10% (b) 12% (c) 15% (d) 18%
51. It is known that 20% of the mangoes are rotten. If number of rotten mangoes is 35, then the total
number of mangoes is:
(a) 150 (b) 175 (c) 180 (d) 185
52. A student has to secure 40% marks to pass. He gets 178 marks and fails by 22 marks. The
maximum marks are?
(a) 200 (b) 500 (c) 800 (d) 1000

53. Find the average of


(a) 0.389 (b) 0.416 (c) 0.260 (d) 0.419

54. Find the average of


(a) 6.50 (b) 81.04 (c) 43.46 (d) 51.49
55. A sum of money is to be distributed among A, B, C, D in the proportion of 5 : 2 : 4 : 3. If C gets Rs.
1000 more than D, what is B's share?
(a) Rs. 500 (b) Rs. 1500 (c) Rs. 2000 (d) Rs. 2500
56. If a:b =15:30 and b:c=30:60, find a:c =?
(a) 1.12 (b) 1.20 (c) 1.25 (d) 1.30
57. The sum of three numbers is 98. If the ratio of the first to second is 2 :3 and that of the second to
the third is 5 : 8, then the second number is:
(a) 20 (b) 30 (c) 48 (d) 58
58. A boy deposited Rs. 300 in a saving bank which pays an interest of 5½%. How much interest will be
get in one year?
(a) Rs. 16.5 (b) Rs. 11 (c) Rs. 15 (d) Rs. 20
59. If x  y = 6 and x + y= 16. Then?
(a) x=5 and y = 3 (b) x = 8 and y = 5 (c) x = 10 and y = 3 (d) x = 11 and y = 5
60. A man walked 3 km towards North, turned West and walked 2 km, then turned North and
walked 1 km and then turned East and walked 5 km. How far is he from his starting point?

Ratio & Proportion Practice


61. If 70 men can build 4 houses in 12 months, how many men would be required to
build 6 houses in 4 months?
(a) 345 men (b) 315 men (c) .301 men (d) 360 men
62. If 40 men build 4 buildings in 12 months, how many men would be required to build
6 houses in 4 months?
(a) 114 men (b) 180 men (c) 140 men (d) 160 men
63. If 80 men dug 4 holes in 12 months, how many men would be required to dig 6 holes
in 4 months?
(a) 140 men (b) 410 men (c) 360 men (d) 40 men
64. If 15 men can do a piece of work in 8 days, how many men will finish it in 10 days?
(a) 14 men (b) 12 men (c) 16 men (d) 8 men
65. If 20 men can do a piece of work in 8 days, how many men will finish it in 10 days?
(a) 4 men (b) 16 men (c) 8 men (d) 18 men
66. If 15 pumps of equal capacity can fill a tank in 7 days, then how many extra pumps
will be required to fill the tank in 5 days?
(a) 14 (b) 7 (c) 6 (d) 21
67. 5 men can dig a big hole in 15 days. How long will it take 2 men to do the work?
(a) Days (b) Days (b) Days (b) Days
68. It takes 14 taps to fill a swimming pool in 10 hours. How long will it take 5 taps to fill
the pool?
(a) 21 (b) 22 (c) 28 (d) 34
69. 8 Machines can do a piece of work in 5 days. How long will it take 4 machines to do
the work?
(a) 21 days (b) 10 days (c) 20 days (d) 8 days
70. A bridge can be constructed by 130 men in 40 days. How long will it take 110 men to
do the work?
(a) 47.27 (b) 41.22 (c) 52.34 (d) 62.21
71. 120 men are used to build huge-building for 30 days. How long will it take 80 men to
do the work?
(a) 35 days (b) 39 days (c) 42days (d) 45 days
72. The bridge is completed in 20 days by 80 men. How long will it take 70 men to
complete the work?
(a) 30 days (b) 29 days (c) 23 days (d) 26 days
73. The road is completed in 35 days by 70 men. How long it will take 40 men to complete
the work?
(a) 58 days (b) 61 days (c) 30 days (d) 32 days.
74. The bridge is completed in 45 days by 105 men. How long it will take 90 men to do the
work.

(a) 68.7 days (b) 61 days (c) days (d) 60 days


75. 6 men can plough a field in 10 hours. How long will it take 4 men to do the work?
(a) 15 hours (b) 10 hours (c) 5 hours (d) 12 hours
76. The sum of three numbers is 98. If the ratio of the first to second is 2 :3 and that of the
second to the third is 5 : 8, second number is:
(a) 20 (b) 30 (c) 48 (d) 58
77. 5 men can dig a big hole in 15 days. How long will it take 2 men to do the work?

(a) 35 days (b) days (c) days (d) days


78. It takes 14 taps to fill a swimming pool in 10 hours. How long will it take 5 taps to fill
the pool?
(a) 21 (b) 22 (c) 28 (d) 34
79. 8 machines can do a piece of work in 5 days. How long will it take 4 machines to do the
work?
(a) 21 days (b) 10 days (c) 20 days (d) 8 days
80. A bridge can be constructed by 130 men in 40 days. How long will it take 110 men to
do the work?
(a) 47.27 (b) 41.22 (c) 52.34 (d) 62.21
81. 120 men are used to build huge building for 30 days. How long will it take 80 men to
do the work?
(a) 35 days (b) 39 days (c) 42 days (d) 45 days
82. 5 men can do a work in 50 days. How long will it take 3 men to complete the work?
(a) 83 days (b) 85 days (c) 90 days (d) 92 days
83. 8 men can do a work in 60 days. How long will it take 6 men to do the work?
(a) 80 days (b) 90 days (c) 120 days (d) 19 days
84. 15 men can do a work in 12 days. How long it will take 9 men to do the work?
(a) 14 days (b) 20 days (c) 9 days (d) 19 days
85. If the cost of a loaf is Rs 5.50. What is the cost of 2½ loaves of bread?
(a) Rs 471.68 (b) Rs 529.76 (c) Rs 13.75 (d) Rs 15.0

86. If the cost of 1 dozen mangoes is Rs 20, what is the cost of dozen mangoes.
(a) Rs 85 (b) Rs 49 (c) Rs 68 (d) Rs 70

87. If the cost of 1 metre fine cloth is Rs 421. What is the cost of metres?
(a) Rs 673.6 (b) Rs 721 (c) Rs 154.80 (d) Rs 522.68

PERCENTAGE QUESTIONS
88. The number, whose 3% is 60, is:
(a) 2000 (b) 3000 (c) 6000 (d) 9000
89. In the exam 45% students failed and 550 students were successful. The total number of students
who appeared in the exam was:
(a) 1000 (b) 900 (c) 1500 (d) 800
90. The original price of a gift pack is Rs.80. A discount of 15% is allowed in shop. What is the sale
price?
(a) Rs. 65 (b) Rs. 68 (c) Rs. 61 (d) Rs. 50
91. What is 15 percent of 34?
(a) 5.10 (b) 4.10 (c) 3.10 (d) 2.10
92. Daniyal sold a chair for Rs.800 which he had bought for Rs.1000. What is his % loss?
(a) 20% (b) 10% (c) 15% (d) 25%
93. Mention the number, whose 17% is 51?
(a) 300 (b) 150 (c) 351 (d) 510
94. In a class of 550 students 42% wish to go to college. How many students wish to attend college?
(a) 23 (b) 77 (c) 210 (d) 231
95. 38 is 20% of what number?
(a) 190 (b) 380 (c) 58 (d) 760
96. A shopkeeper buys an article for Rs. 27 and sells it at a profit of 10 percent of the selling price.
What is the selling price of the article?
(a) Rs.30 (b) Rs.35 (c) Rs. 29.50 (d) Rs.29
97. The price of a car including 6 percent sales tax is Rs. 154,431.40. What is the market price of the
car-without tax?
(a) 145,690 (b) 150,000 (c) 145,500 (d) 145,000
98. 18 is 75% of ____________.
(a) 24 (b) 25 (c) 34 (d) 32
99. 25 Students took a test and 4 of them failed. What percent of them passed the test?
(a) 75% (b) 80% (c) 82% (d) 84%
100. A man buys a toy for Rs. 70 after getting a discount of 20%. What was the marked price of the
toy?
(a) Rs:84 (b) Rs:56 (c) Rs: 87.50 (d) Rs:90
101. In a class of 550 students, 42% wish to go to college. How many wish to attend college?
(a) 200 (b) 210 (c) 231 (d) 252
102. A toy is sold for Rs. 220 What was the cost if the rate of profit was 10% of the cost?
(a) Rs. 196 (b) Rs. 198 (c) Rs. 200 (d) Rs. 210
103. The number whose 17% is 51 is:
(a) 200 (b) 100 (c) 300 (d) 210
104. Ahmed spend Rs.44,000 to get a car and sells it at Rs.48,000. Find the percentage of
profit?
(a) 12% (b) 9.09% (c) 14% (d) 80%
105. Sohail Khan sold a car for Rs.80,000 which he was bought for Rs.90,000. What is his
loss?
(a) 61:40% (b) 55.51% (c) 12.31% (d) 11.11%
106. Ali gained 510/850 marks in matriculation examination. What percentage of marks did he gain?
(a) 60% (b) 57% (c) 71% (d) 73%
107. Two employees X and Y are paid a total of Rs. 550 per week by their employer. If X is
paid 120 percent of the sum paid to Y, how much is Y paid per week?
(a) Rs.120 (b) Rs.250 (c) Rs.280 (d) Rs.320
108. Ahmed spends Rs.400 in getting a good table and sells it at Rs.600. Find the %age of profit?
(a) 90% (b) 45% (c) 50% (d) 80%
109. Ali spends Rs.500 to get a good shirt and sells it at Rs.800. Find the % age of profit?
(a) 59% (b) 60% (c) 63% (d) 40%
110. Ahmed spends a Rs.44,000 to get a car and sells it at Rs.48,000: Find its % in profit?
(a) 12% (b) 9.09% (c) 14% (d) 80%
111. Kiran spends Rs.33,000 to get a new computer. After few days, the price of computer rises and she
sells it for Rs.35, 000. How much profit, did she get in %age?
(a) 82% (b) 90% (c) 46% (d) 6.06%
112. Saima scored 178 marks out of 300 marks in a certain text. What percentage of marks did she
score?
(a) 48% (b) 52.41% (c) 61.41 % (d) 59.33%
113. The total distance from school to house is 35km. Arif covered a distance of 17km. What
percentage of the whole distance has he covered?
(a) 31% (b) 21% (c) 48.57% (d) 57%
114. This road is only 50km long. We covered a distance of 20km. What percentage of distance have we
covered?
(a) 40% (b) 69% (c) 48% (d) 53%
115. We covered a 150km distance of Islamabad road. The total distance is 370km. What percentage of
distance have we covered
(a) 30% (b) 40.54% (c) 67% (d) 41%
116. Ali gained 510 marks in matriculation examination out of 850 marks. What percentage of marks
did he gain?
(a) 60% (b) 57% (c) 71 % (d) 73%
117. The whole journey in a walking competition is 15km. Ali has completed 9km. What percentage of
the whole journey has he completed?
(a) 60% (b) 70% (c) 75% (d) 64%
118. The total distance for a long walk is 25km. Asim covered a distance of 10km. What percentage of
the whole walk has he completed?
(a) 60% (b) 40% (c) 70% (d) 50%
119. If 30 % of a number is 12.6, then the number is:
(a) 41 (b) 51 (c) 52 (d) 42
120. What percentage of 84cm is 15cm?
(a) 21 % (b) 32% (c) 44% (d) 17.86%
121. The price of a TV set increases from Rs.4500 to Rs.8000, what is the percentage increase?
(a) 77.78% (b) 90% (c) 40% (d) 14%
122. The price of a book increases from Rs.120 to Rs.150. What is the percentage increases?
(a) 35% (b) 25% (c) 15% (d) 251 %
123. The price of a glass set increases from Rs.600 to Rs.710 What is the percentage increases?
(a) 18.33% (b) 17.33% (c) 10% (d) 14%
124. (30% of 80)/? =24
(a) 3/10 (b) 3/17 (c) 1 (d) 2
125. 45% of ? + 30% of 90 = 30% of 210.
(a) 120 (b) 80 (c) 60 (d) 90
126. One-fourth of one-third of two-fifth of a number is 15. What will be 40% of that number?
(a) 120 (b) 350 (c) 270 (d) 180
127. If 37% of a number is 990.86, what will be approximately 19% of that number?
(a) 600 (b) 400 (c) 500 (d) 700
128. 35% of 30=25% of ? +1
(a) 28 (b) 38 (c) 42 (d) 32
129. If 15% of 40 is greater than 25% of a number by 2, the number is?
(a) 16 (b) 20 (c) 24 (d) 32
130. What will be 160% of a number whose 200% is 140?
(a) 112 (b) 160 (c) 1401 (d) 200
131. If 75% of a number is added to 75, the result is the number itself. Then, the number is:
(a) 400 (b) 300 (c) 60 (d) 50
132. Subtracting 40% of a number from the number, we get the result as 30. The number is:
(a) 28 (b) 50 (c) 52 (d) 70
133. Subtracting 6% of x from x is equivalent to multiplying x by how much?
(a) 0.94 (b) 9.4 (c) 0.094 (d) 94
134. If 8% of x=4% of y, then 20% of x is:
(a) 10% of y (b) 16% of y (c) 80% of y (d) None of these
135. An estimated 3 out of every 25 men are left-handed. What percent of men are left-handed?
(a) 10% (b) 12% (c) 15% (d) 18%
136. It is known that 20% of the mangoes are rotten. If number of rotten mangoes is 35, then the total
number of mangoes is:
(a) 150 (b) 175 (c) 180 (d) 185
137. A student has to secure 40% marks to pass. He gets 178 marks and fails by 22 marks. The
maximum marks are?
(a) 200 (b) 500 (c) 800 (d) 1000
138. Albert buys 4 horses and 9 cows for Rs. 13,400. If he sells the horses at 10% profit and the cows at
20% profit, then he earns a total profit of Rs.1880. The cost of a horse is:
(a) Rs. 2200 (b) Rs. 2000 (c) Rs. 2700 (d) Rs. 3200
139. The C.P. of two watches taken together is Rs. 840. If by selling one at a profit of 16% and the other
at a loss of 12%, there is no loss or gain in the whole transaction, then the C.P of the two watches
respectively is:
(a) Rs. 360, Rs. 480 (b) Rs. 500, Rs. 360
(c) Rs. 360, Rs. 460 (d) Rs. 400, Rs. 440
140. A boy deposited Rs. 300 in a saving bank which pays an interest of 5½%. How much interest will
be get in one year?
(a) Rs. 16.5 (b) Rs. 11 (c) Rs. 15 (d) Rs. 20
141. A woman deposited Rs. 500 in a saving bank which pays an interest of 5½%. How much interest
will she get in one year?
(a) 41 Rs (b) 27.5 Rs (c) 31 Rs (d) 34 Rs
142. A man deposited Rs. 1500in a saving bank which pays an interest of 3½%. How much interest will
he get in one year?
(a) 41.49 Rs (b) 24.60 Rs (c) 52.5 Rs (d) 25.6 Rs
143. A man deposited Rs. 2500 in a saving bank which pays an interest of 3½%. How much interest will
he get in one year?
(a) 87.5 Rs (b) 17.5 Rs (c) 25.41 Rs (d) 46.87 Rs
144. Find the interest on Rs.300 for 2½ years at 4% per year? What is the amount at the end of 2½
years?
(a) Rs.40, Rs.300 (b) Rs.l0. Rs.400 (c) Rs.25, Rs.280 (d) Rs.30, Rs.330
145. A tailor spends Rs. 200 in getting on "Awami suit" prepared and sells the same for Rs. 300. Find
his profit, in percent?
(a) 50% (b) 40% (c) 70% (d) 80%
146. Shani bought a table for Rs.200 and sold it Rs.175. How much loss did he get?
(a) Rs. 25 (b) Rs. 40 (c) Rs. 410 (d) Rs. 200
147. Rashid bought a shirt for Rs.350 and sold it Rs.300. How much loss did he get?
(a) Rs. 40 (b) Rs. 50 (c) Rs. 20 (d) Rs. 80
148. If Rs. 1000 are invested at 12% interest and interest is compounded half-yearly, what will be the
total amount at the end of one year?
(a) Rs. 1120.00 (b) Rs.1123.60 (c) Rs. 1126.20 (d) Rs. 1127.20
149. Ten percent of twenty plus twenty per cent of ten equals:
(a) 2 percent of 200 (b) 20 percent of 200 (c) 1 percent of 200 (d) 10 percent of 200

150. If 40% of a number is equal to two-third of another number, what is the ratio of first number to
the second number?
(a) 2 : 5 (b) 3:7 (c) 5:3 (d) 7:3
151. Ali sold a table Rs.800 which he was bought for Rs.1000. What is his % loss?
(a) 20% (b) 60% (c) 14% (d) 17%
152. Akber sold his old gun Rs.900 which he was bought for Rs.2500. What is his % loss?
(a) 64% (b) 82% (c) 94% (d) 15%
153. Ali sold his old car Rs.43,000. He was bought it for Rs.50,000 What is his percentage Loss?
(a) 14% (b) 16% (c) 17% (d) 28%
154. Shani sold an old computer Rs.22,000 and he was bought it for 27.000. What is his loss?
(a) 32.37% (b) 25.92% (c) 34.15 (d) 18.51%
155. Rohail sold a car Rs.80,000 which he was bought for Rs.90,000. What is his % loss?
(a) 61.40% (b) 55.51 (c) 12.31% (d) 11.11
156. A man's basic pay for a 40-hour week is Rs. 2000. Overtime is paid for at 25% above the basic
rate. In a certain week, he worked overtime and his total wage was Rs. 2500. He, therefore, worked
for a total of:
(a) 45 hours (b) 47 Hours (c) 48 hours (d) 50 hours
157. A primary school had an enrolment of 850 pupils in January 1970. In January 1980, the enrolment
was 1,120; What was the percentage increase for the enrolment?
(a) 31.76%' (b) 33.50% (c) 30,65% (d) 34.76%
158. Mr. Rehan ordered a car worth Rs.6, 00.000 and was given a discount. Given that he paid
Rs.5,70,000 for his new car, calculate the percentage discount he received:
(a) 5% (b) 7% (c) 9% (d) 10%
159. Naveed spends Rs. 400 in getting a good table and sells it at Rs.600. Find the percentage of profit?
(a) 90% (b) 45% (c) 50% (d) 80%
160. Ali spends Rs. 500 to get a good shirt and sells it at Rs.800. Find the percentage of Profit?
(a) 59% (b) 60% (c) 63% (d) 40%
161. A man saves Rs. 500 which is 15% of his annual income. How much does he earn in one year?
(a) 3542.50 (b) 3333.33 (c) 3132.30 (d) 3075.75
162. What simple interest rate will Susan need to secure to make $2,500 in interest on a $10,000
principal over 5 years?
(a) 4% (b) 5% (c) 6% (d) 7%
163. Ali sold a table Rs.800 which he was bought for Rs.1000. What is his % loss?
(a) 20% (b) 60% (c) 14% (d) 17%
164. A basketball team won 18 out of 30 games played. What percentage of games did the team win?
(a) 60% (b) 40% (c) 43% (d) 87%
165. Arif gained 8 marks in test of 20 marks. What percentage of marks did he gain?
(a) 48% (b) 80% (c) 72% (d) 40%
166. Ali gained 18 marks in monthly test consisting of 50 marks what percentage of marks he gained?
(a) 51 % (b) 36% (c) 48% (d) 64%
167. A covered 8km distance out of 1000o. What percentage of distance did he covered?
(a) 51.89% (b) 80% (c) 67% (d) 41
168. What percentage of 2m is 10 cm?
(a) 5% (b) 8% (c) 10% (d) 15%
169. Three candidates contested an election and received 1136, 7636 and 11628 votes respectively. What
percentage of the total votes did the winning candidate get?
(a) 57% (b) 60% (c) 65% (d) 90%
170. The length and breadth of a square are increased by 40% and 30% respectively. The area of the
resulting rectangle exceeds the area of the square by:
(a) 82% (b) 42% (c) 62% (d) None of these
171. The length of a rectangle is increased by 20% and the width is decreased by 20%. The area
decreases by:
(a) 0.8% (b) 1.2% (c) 4% (d) 8%
172. The length of a rectangle is increased by 60%. By what percent would the width have to be
decreased to maintain the same area?
(a) 75% (b) 37.5% (c) 60% (d) None of these
173. 218% of 1674 = ? x 1800
(a) 4 (b) 0.5 (c) 6 (d) None of these
174. 63% of 34/7 is?
(a) 2.25 (b) 2.40 (c) 2.50 (d) 3.06
175. If 30 % of a number is 12.6, then the number is?
(a) 41 (b) 51 (c) 52 (d) 42
176. 5% of (25% of Rs. 1600) is?
(a) Rs. 5 (b) Rs. 17.50 (c) Rs. 20 (d) Rs. 25
177. (25/3) % of ? = 150
(a) 1250 (b) 1800 (c) 1700 (d) 1400
178. A number is increased by 20% and then decreased by 20%, the final value of the number.
(a) Decreases by 4% (b) Increases by 4%
(c) Decreases by 2% (d) Decrease not change
179. Ali receives a commission of Rs. 25 for every Rs. 250 worth of merchandise he sells. The
percentage of his commission is:
(a) 10% (b) 25% (c) 15% (d) 5%
180. Rashid drew a square, he then erased it and drew a second square whose sides were 3 times the
side of the first square. By what percent was the area of the square increased?
(a) 300% (b) 800% (c) 400% (d) None of these
181. A man buys shirts at Rs. 80 each and sells them at Rs. 100 each. What percentage of profit does he
make on the sale of each?
(a) 28% (b) 40% (c) 25% (d) 80%
182. A man buys calculators at Rs. 120 each and sells them at Rs.150 each. What percentage of profit
does he make on the sale of each calculator?
(a) 50% (b) 30% (c) 10% (d) 25%
183. Ahmed spends Rs.400 in getting a good table and sells it for Rs.600. Find the %age profit.
(a) 90% (b) 45% (c) 50% (d) 80%
184. A seller marks his goods 30% above the cost price but allows a discount of 15% from
the marked price. His percentage of profit is:
(a) 10.5 (b) 15 (c) 9 (d) 8.5
185. Find 250% of 36:
(a) 90 (b) 80 (c) 60 (d) 20
186. A shopkeeper bought bicycle for R.s 600 and sold it for R.s 720. find the gain percent.
(a) 15% (b) 18% (c) 20% (d) 25
187. What simple interest rate will Susan need to secure to make $2,500 in interest on a $10,000
principal over 5 years?
(a) 4% (b) 5% (c) 6% (d) 7%

BASIC ARITHMETIC
1. For how many positive integers, a, is it true that a2 ≤ 2 a2
(a) None (b) 1 (c) 3 (d) 4
(e) More than 4
2. If 0 < a < b < 1, which of the following statements are true?
Indicate all such statement.

(a) a – b is negative (b) is positive (c) a is positive


3. If the product of 4 consecutive integers is equal to one of them, what is the large possible value of
one of the integers?

4. At 3:00 A.M. the temperature was 13° below zero. By soon it had risen to 32° . What was the
average hourly increase in temperature?

(a) (b) (c) 5° (d) 7.5°


(e) 45°
5. If a and b are negative, and c is positive, which of the following statements are true? Indicate all
such statements.

(a) (b) (c)


6. If – 7 ≤ x ≤ 7 and 0 ≤ y ≤ 12, what is the greatest possible value of y- x?
(a) –19 (b) 5 (c) 7 (d) 17
(e) 19

7. If (7a) (7b) = what is d in terms of a, b, and c?

(a) (b) (c) (d)

(e)
8. If each of and  can be replaced by +, , or ×, how many different values are there for the
expression 2 2 ?

9. A number is “terrific” if it is a multiple of 2 or 3. How many terrific numbers are there between
11 and 11?
(a) 6 (b) 7 (c) 11 (d) 15
(e) 17
10. If x  y represents the numbers of integers greater than x and less than y, what is the value of 

(a) 2 (b) 3 (c) 4 (d) 5
(e) 6
Questions 11 and 12 refer to the following definition
For any positive integer n,  (n) represents the numbers of positive divisors of n.
11. Which of the following statements are true? Indicate all such statements.
(a) (b) (c)
12. What is the value of  ( ( (12)))?
(a) 1 (b) 2 (c) 3 (d) 4
(e) 6
13. If p and q are primes greater than 2, which of the following statements must be true?
Indicate all such statements.
(a) p + q is even (b) pq is odd (c) p2 – q2 is even
14. If 0 < x <1, which of the following lists the numbers in increasing order?
(a) (b) (c) (d)
8 9 10
15. Which of the following is equal to (7 x7 ) ?
(a) 7 27 (b) 7 82 (c) 7 170 (d) 49 170
720
(e) 49
Answer Key
1. c 2. a, b 3. 3 4. c 5. b, c
6. e 7. b 8. 4 9. d 10. d

FRACTION AND DECIMALS


1. A biology class has 12 boys and 18 girls. What fraction of the class are boys?

16. For how many integers, a, between 30 and 40 is it true that , and are all in lowest terms?
(a) 1 (b) 2 (c) 3 (d) 4
(e) 5
17. What fractional part of a week is 98 hours?

18. What is the value of the product

(a) (b) (c) (d)

(e)

19. If of a number is 22, what is of that number?


(a) 6 (b) 11 (c) 12 (d) 33
(e) 44

20. Jason won some goldfish at the state fair. During the first week, of them died, and during the

second week, of those still alive at the end of the first week died. What fraction of the original
goldfish were still alive after two weeks?

(a) (b) (c) (d)

(e)

21. of 24 is equal to of what number?


(a) 7 (b) 8 (c) 15 (d)

(e)

22. If 7a = 3 and 3b = 7, what is the value of ?

(a) (b) (c) 1 (d)

(e)

23. What is the value of ?

(a) (b) (c) (d)

(e)

24. Which of the following expressions are greater than x when x = ?


Indicate all such expression.

(a) (b) (c)

25. One day at Lincoln High School, of the students were absent, and of those present went on
a field trip. If the number of students staying in school that day was 704, how many students are
enrolled at Lincoln High?

26. If a = 0.87, which of the following expressions are less than a?


Indicate all such expression.

(a) (b) (c)

27. For what value of x is

?
(a) .001 (b) .01 (c) .1 (d) 10
(e) 100
28. If A = { 1, 2, 3}, B = {2, 3, 4} and C is the set consisting of all the fraction whose numerators are in
A and whose denominators are in B, what is the product of all of the numbers in C?

(a) (b) (c) (d)

(e)
29. For the final step in a calculation, Ezra accidentally divided by 1000 instead of multiplying by
1000. What should he do to his incorrect answer to correct it?
(a) Multiply it by 1000. (b) Multiply it by 100,000.
(c) Multiply it by 1,000,000. (d) Square it
(e) Double it
Answer key
1. 2. c 3. 4. a 5. e

6. c 7. a 8. a 9. a 10. a, b
11. 960 12. b 13. d 14. a 15. c

Practice Exercise — Percents

Discrete Quantitative Questions


1. If 25 students took an exam and 4 of them failed, what percent of them passed?
(a) 4% (b) 21% (c) 42% (d) 84%
(e) 96%
1. Amanda bought a $60 sweater on sale at 5% off. How much did she pay, including 5% sales tax?
(a) $54.15 (b) $57.00 (c) $57.75 (d) $59.85
(e) $60.00
2. What is 10% of 20% of 30%?
(a) 0.006% (b) 0.6% (c) 6% (d) 60%
(e) 6000%
3. If c is a positive number, 500% of c is what percent of 500c?
(a) 0.01 (b) 0.1 (c) 1 (d) 10
(e) 100
4. What percent of 50 is b?

(a) (b) (c) (d)


(e) 2b

5. 8 is % of what number?

(e) 1%
6. There are twice as many girls as boys in an English class. If 30% of the girls and 45% of the boys
have already handed in their book reports, what percent of the students have not yet handed in
their reports?
%
Answer Key
1. d 2. d 3. b 4. c 5. e
6. 2400 7. c 8. d 9. b 10. b
11. d 12. 65 13. d 14. 150 15. d
Ratios and Proportions

Discrete Quantitative Questions

1. If of the employees in a supermarket are not college graduates, what it is the ratio of the number
of college graduates to those who are not college graduates?
(a) 1:3 (b) 3:7 (c) 3:4 (d) 4:3
(e) 3:1

2. If , what is the value of a2 ?


(a) 3 (b) 3 (c) 9 (d) 45
(e) 90
3. If 80% of the applicants to a program were rejected, what is the ration of the number accepted to
the number rejected?

4. Scott can read 50 pages per hour. At this rate, how many pages can he read in 50 minutes?

(a) 25 (b) (c) (d) 48


(e) 60
5. If all members of a team are juniors or seniors, and if the ratio of junior to seniors on the team is
3:5, what percent of the team members are seniors?
(a) 37.5% (b) 40% (c) 60% (d) 62.5%
(e) it cannot be determined from the information given.
6. The measures of the three angles in a triangle are in ratio of 1: 1:2. Which of the following
statements must be true?
Indicate all such statements
(a) The triangle is isosceles (b) The triangle is a right triangle
(c) The triangle is equilateral.
7. What is the ratio of the circumference of a circle to its radius?

(a) 1 (b) (c) (d)


(e)
8. The ratio of the number of freshmen to sophomores to juniors to seniors on a college basketball
team is 4:7:6:8. What percent of the team are sophomores?
(a) 16% (b) 24% (c) 25% (d) 28%
(e) 32 %
9. At central state college the ratio of the number of students taking Spanish to the number taking
French is 7:2. If 140 students are taking French, how many are taking Spanish?

stu d en ts

10. If a:b = 3:5 a:c = 5:7, what is the value of b:c?


(a) 3:7 (b) 21:35 (c) 21:25 (d) 25:21
(e) 7:3

11. If x is a positive number and then x =


(a) 3 (b) 4 (c) 6 (d) 12
(e) 36
12. In the diagram, below, b:a =a 7:2. What is b – a?
b a

(a) 20 (b) 70 (c) 100 (d) 110


(e) 160
13. A snail can move i inch in m minutes. At this rate, how many feet can it move in h hours?

(a) (b) (c) (d)


(e)

14. Gilda can grade t tests in hours. At this that how many tests can she grade in x hours?

(a) (b) (c) (d)

(e)
15. A club had 3 boys and 5 girls. During a membership drive the same number of boys and girls
joined the club. How many member does the club have now if the ratio of boys to girls is 3:4?
(a) 12 (b) 14 (c) 16 (d) 21
(e) 28

16. If what is the value of x?

(a) (b) 3 (c) 7 (d) 17


(e) 136
17. If 4 boys can shovel a driveway in 2 hours, how many minutes will it take 5 boys to do the job?
(a) 60 (b) 72 (c) 96 (d) 120
(e) 150
18. If 500 pounds of much will feed 20 pigs of a week, for how many days will 200 pounds of much
feed 14 pigs?

Answer Key
1. a 2. d 3. 4. b 5. d

6 a, b 7 e 8. d 9. 490 10. d
11. c 12. c 13. a 14. b 15. b
16. d 17. c 18 4
Averages
1. Michael’s average (arithmetic mean) on 4 tests in 80. What does he need on his fifth test to raise
his average to 84?
(a) 82 (b) 84 (c) 92 (d) 96
(e) 100
2. Maryline’s average (arithmetic mean) on 4 tests is 80. Assuming she can earn no more than 100 on
any test, what is the least she can earn on her fifth test and still have a chance for an 85 average,
after even tests?
(a) 60 (b) 70 (c) 75 (d) 80
(e) 85
3. Sandrine’s average(arithmetic mean) on 4 tests is 80. Which of ht following cannot be the number
of tests on which she earned exactly 80 points?
(a) 0 (b) 1 (c) 2 (d) 3
(e) 4
4. What is the average (arithmetic mean) of the positive integers from 1 to 100, inclusive?
(a) 49 (b) 49.5 (c) 50 (d) 50.5
(e) 51
5. If 10a + b10 = 35, what is the average (arithmetic mean) of a and b?

6. If x + y = 6, y + z = 7, and z + x = 9 what is the average (arithmetic mean) of x, y, and z?

(a) (b) (c) (d) 11


(e) 22
7. If the average (arithmetic mean) of 5, 6, 7, and w is 8, what is the value of w?
(a) 8 (b) 12 (c) 14 (d) 16
(e) 24
8. What is the average (arithmetic mean) in degrees of the measure of the five angles in a pentagon?
d eg ree s

9. If a + b = 3 (c + d), which of the following is the average (arithmetic mean) of a, b, c and d?

(a) (b) (c) (d)


(e) c + d
10. In the diagram below, lines and m are not parallel.

a b
d c

e f 
h g

If A represents the average (arithmetic mean) of the degree measure of all eight angles, which of
the following is true?
(a) A = 45 (b) 45 < A < 90 (c) A = 90 (d) 90 < A < 180
(e) A = 180
11. What is the average (arithmetic mean) of 210 and 220?
(a) 2 15 (b) 25 + 210 (c) 29 + 219 (d) 229
(e) 30
12. Let M be the median and m the mode of the following set of numbers: 10, 70, 20, 40, 70, 90. What is
the average (arithmetic mean) of M and m?
(a) 50 (b) 55 (c) 60 (d) 62.5
(e) 65
Answer key
1. e 2. c 3. d 4. d 5. 1.75
6. a 7. c 8. 108 9. e 10. c
11. c 12. d
POLYNOMIALS
Discrete Quantitative Questions

1. What is the value of when a = 117 and b = 118?

2. If a2 – b2 = 21 and a2 + b2 = 29, which of the following could be the value of ab?


Indicate all possible values.
(a) –10 (b) 5 (c) 10
3. What is the average (arithmetic mean) of x + 2x – 3, 3x2 – 2x – 3, and 30 – 4x2
2

(a) (b) (c) (d) –12


(e) 8
4. What is the value of x2 + 12x + 36 when x = 994?
(a) 11,928 (b) 99,836 (c) 100,000 (d) 988,036
(e) 1,000,000
5. If c2 + d 2 = 4 and (c – d) 2 = 2, what is the value of cd?
(a) 1 (b) (c) 2 (d) 3
(e) 4
6. What is the value of
(2x + 3) (x + 6) – (2x – 5) (x + 10)?
(a) 32 (b) 16 (c) 68 (d) 4x2 + 30x + 68
(e) 4x2 + 30x – 32

7. If and ab = c, what is the average of a and b?

(a) 0 (b) (c) 1 (d)

(e)
8. If x2 – y2 = 28 and x – y= 8, what is the average of x and y?
(a) 1.75 (b) 3.5 (c) 7 (d) 8
(e) 10
9. Which of the following is equal to

(a) 0 (b) 4 (c) (d)

(e)

10. If = 100, what is the value of .


Answer Key
1. 235 2. a,c 3. e 4. e 5. a
6. c 7. b 8. a 9. b 10. c
EQUATIONS
1. If 4x + 12 = 36, what is the value of x+ 3?
(a) 3 (b) 6 (c) 9 (d) 12
(e) 18
2. If 7x +10 = 44, what is the value of 7x – 10?

(a) (b) (c) (d)


(e) 34

3. If 4x + 13 = 7 – 2x, what is the value of x?

(a) (b) (c) (d)

(e)
4. If x – 4 = 9, what is the value of x2 – 4?

5. If ax – b = c – dx, what is the value of x in terms of a, b, c, and d?

(a) (b) (c) (d)

(e)

6. If , what is the value of x?


(a) 3 (b) 18 (c) 27 (d) 54
(e) 72
7. If 3x – 4 = 11, what is the value of (3x – 4)2?
(a) 22 (b) 36 (c) 116 (d) 121
(e) 256
8. If 6412 = 2a – 3, what is the value of a?
(a) 9 (b) 15 (c) 69 (d) 72
(e) 75
9. 4b is less than 50, and a is twice b, what is the largest possible integer value of a?
(a) 9 (b) 10 (c) 11 (d) 19
(e) 20

10. If , then a =

(a) b + 5 (b) b – 5 (c) b + (d) b –

(e)
11. If x = 3a + 7 and y = 9a2, what is y in terms of x?

(a) (b) 3 (c) (d)

(e)
12. If 4y – 3x = 5, what is the smallest integer value of x for which y > 100?
Answer Key

1. c 2. d 3. c 4. 165 5. a

6. d 7. d 8. e 9. d 10. c

11. a 12. 132

Practice Exercise — Word Problems

Discrete Quantitative Questions


1. Howard has three times as much money as Ronald. If Howard gives Ronald $50, Ronald will then
have three times as much money as Howard. How much money, in dollars, do the two of them
have together?
d o llars

2. In the afternoon, Beth read 100 pages at the rate of 60 pages per hour; in the evening, when she
was tired, she read another 100 pages at the rate of 40 pages per hour. What was her average rate
of reading for the day?
(a) 45 (b) 48 (c) 50 (d) 52
(e) 55
3. If the sum of five consecutive integers is S, what is the largest of those integers in terms of S?

(a) (b) (c) (d)

(e)
4. As a fund-raiser, the school band selling two types of candy: lollipops for 40 cents each and
chocolate bars for 75 cents each. On Monday, they sold 150 candies and raised 74 dollars. How
many lollipops did they sell?
(a) 75 (b) 90 (c) 96 (d) 110
(e) 120

5. A jar contains only red, white, and blue marbles. The number of red marbles is the numbers of

white ones, and the number of white ones is the numbers of blues ones. If there are 470 marbles
in all, how many of them are blue?
(a) 120 (b) 135 (c) 150 (d) 184
(e) 200
6. The numbers of shells in Judy’s collections is 80% of the number in Justin’s collections. If Justin
has 80 more shells than Judy, how many shells do they have altogether?

7. What is the greater of two numbers whose production is 900, if the sum of the two numbers
exceeds their difference by 30?
(a) 15 (b) 60 (c) 75 (d) 90
(e) 100
8. On a certain project the only grades, awarded were 80 and 100. If 10 students completed the
project and the average of their grades was 94, how many earned 100?
(a) 2 (b) 3 (c) 5 (d) 7
(e) 8

9. If years ago Adam was 12, and years from now he will be 2x years old, how old will he be
3x years from now?
(a) 18 (b) 24 (c) 30 (d) 54
(e) It cannot be determined from the information given.
10. Since 1950, when Barry was discharged from the army, he was gained 2 pounds every year. In
1980 he was 40% heavier than in 1950. What percent of his 19954 weight as his 1980 weight?
(a) 80 (b) 85 (c) 87.5 (d) 90
(e) 95
Answer Key

1. 100 2. b 3 e 4 d 5 e

6 720 7 b 8 d 9 d 10 c

Lines and Angles


1. In the figure below, what is the average (arithmetic mean) of the measure of the five angles?

(a) 36 (b) 45 (c) 60 (d) 72


(e) 90

2. In the figure below, what is the value of ?

(a) 1 (b) 10 (c) 11 (d) 30


(e) 36
3. In the figure below, what is the value of b?

4. In the figure below, what is the value of x if y:x = 3:2?


(a) 18 (b) 27 (c) 36 (d) 45
(e) 54
5. What is the measure, in degrees, of the angle formed by the minutes and hour hands of a clock at
1:50?

6. Concerning the figure below, if a = b, which of the following statements must be true?
Indicate all such statements.

(a) c = d (b) ℓ and k are parallel


(c) m and ℓ are perpendicular
7. In the figure below, a:b = 3:5 and c:b = 2:1. What is the measure of the largest angle?
(a) 30 (b) 45 (c) 50 (d) 90
(e) 100
8. A, B, and C are points on a line B between A and C. Let M and N be the midpoints of AB and BC,
respectively. If AB:BC = 3:1, what is MN:BC?
(a) 1:2 (b) 2:3 (c) 1:1 (d) 3:2
(e) 2:1
9. ???????
What is the value of y – x?
(a) 15 (b) 30 (c) 45 (d) 60
(e) 75
10. ??????
And line is ℓ bisects AOB. What is the measure of DOE?
(a) 75 (b) 90 (c) 100 (d) 105
(e) 120
Answer Key

1. d 2. c 3. 36 4. c 5. 115

6. a 7. e 8. e 9. c 10. 8

Triangles
Discrete Quantitative Questions

1. In the angles above, what is the value of x?


(a) 20 (b) 30 (c) 40 (d) 50
(e) 60
2. If the difference between the measure of the two smaller angles of a right triangle is 8°. What is the
measure, in degree, of the smallest angles?
(a) 37 (b) 41 (c) 42 (d) 49
(e) 53
3. What is the area of the equilateral triangle whose altitude is 6?
(a) 18 (b) (c) (d) 36
(e)
4. Two sides of a right triangle are 12 and 13. Which of the following could be the length of the third
side?
Indicate all possible lengths.
(a) 2 (b) 5 (c) (d) 11
(e)
5. What is the value of PS in the triangles above?
(a) (b) 10 (c) 11 (d) 13
(e)
6. If the measure of the angles of a triangle are in the ratio of 1:2:3, and if the length of the smallest
side of the triangle is 10, what is the length of the longest side?
(a) (b) (c) 15 (d) 20
(e) 30

7. What is the value of x in the figure above?

8. In the figure above, what is the value of w?

Questions 9 – 10 refer to the following figure.

box
9. What is the area of ∆BED?
(a) 12 (b) 24 (c) 36 (d) 48
(e) 60
10. What is the perimeter of ∆BED?
(a) 19 + 5 (b) 28 (c) 17 + (d) 32
(e) 36
Questions 11 – 12 refer to the following figure.
box
11. What is the area of ∆DFH?
(a) 3 (b) 4.5 (c) 6 (d) 7.5
(e) 10
12. What is the perimeter of ∆DFH?
(a) 8 + (b) 8 + (c) 16 (d) 17
(e) 18
13. Which of the following expresses true relationship between x and y in the figure above?
(a) y = 60 – x (b) y = x (c) x + y = 90 (d) y = 180 – 3x
(e) x = 90 – 3y
Questions 14 – 15 refer to following figure.
14. What is the perimeter of ∆ABC?
(a) 48 (b) 48 + 12 (c) 48 + 12 (d) 60
(e) 60 + 6
15. What is the area of ∆ABC?
(a) 108 (b) 54 + 72 (c) 54 + 72 (d) 198
(e) 216
Answer Key

1. d 2. b 3. b 4. b,e 5. d

6. 115 7. 110 8. b 9. d 10. d

11. b 12. b 13. a 14. c 15. c


QUADLATRAL

Discrete Quantitative Questions


1. If the length of a rectangle is 4 times its width, and if its area is 144, what is its perimeter?

Questions 2–3 refer to the diagram below in which the diagonals of square ABCD intersect at E.
2. What is the area of ∆DEC?

(a) (b) 1 (c) (d) 2


(e)
3. What is the perimeter of ∆DEC?
(a) 1 + (b) 2 + (c) 4 (d) 2 + 2
(e) 6
4. If the angles of a five- sided polygon are in the ratio of 2:3:3:5:5, what is measure of the smallest
angle?
(a) 20 (b) 40 (c) 60 (d) 80
(e) 90
5. If in the figures below, the area of rectangle ABCD is 100, what is the area of rectangle EFGH?

(a) 98 (b) 100 (c) 102 (d) 104


(e) 106
Questions 6–7 refer to a rectangle in which the length of each diagonal is 12, and one of the angles
formed by the diagonal and a side measures 30°.
6. What is the area of the rectangle?
(a) 18 (b) 72 (c) 18 (d) 36
(e) 36
7. What is the perimeter of the rectangle?
(a) 18 (b) 24 (c) 12 + 12 (d) 18 + 6
(e) 24
8. How many sides does a polygon have if the measure of each interior angle is 8 times the measure of
each exterior angle?
(a) 8 (b) 9 (c) 10 (d) 12
(e) 18
9. The length of a rectangle is 5 more than the side of a square, and the width of the rectangle is 5 less
than the side of the square. If the area of the square is 45, what is the area of the rectangle?
(a) 20 (b) 25 (c) 45 (d) 50
(e) 70
Question 10–11 refer to the following figure, in which M, N, O and P are the midpoints of the sides
of rectangle ABCD.
box
10. What is the perimeter of quadrilateral MNOP?
(a) 24 (b) 32 (c) 40 (d) 48
(e) 60
11. What is the area of quadrilateral MNOP?
box
12. In the figure above, what is the sum of the measure of all of the marked angles?
(a) 360 (b) 540 (c) 720 (d) 900
(e) 1080
13. In quadrilateral WXYZ, the measure of angle Z is 10 more than twice the average of the measure
of the other three angles. What is the measure of angle Z?
(a) 100 (b) 105 (c) 120 (d) 135
(e) 150
Questions 14–15 refer to the following figure, in which M and N are the midpoints of two of the
sides of square ABCD.
14. What is the perimeter of the shaded region?
(a) 3 (b) 2 + 3 (c) 3 + 2 (d) 5
(e) 8
15. What is the area of the shaded region?
(a) 1.5 (b) 1.75 (c) 3 (d) 2
(e) 3
Answer Key
1. 60 2. b 3. d 4. c 5. c

6. d 7. c 8. e 9. a 10. c

11. 96 12. c 13. 150 14. b 15. a

PRACTICE EXERCISE — CIRCLES

Discrete Quantitative Questions


1. What is the circumference of a circle whose area is 100?
(a) 10 (b) 20 (c) 10 (d) 20
(e) 25
2. What is the area of a circle whose circumference is

(a) (b) (c) (d)


(e)
3. What is the area of a circle that is inscribed in a square of area of 2?

(a) (b) (c) (d)


(e)
4. A square of area 2 inscribed in a circle what is the area of the circle?

(a) (b) (c) (d)


(e)
5. A 5  12 rectangle is inscribed in a circle what is the radius of ht circle?
box
6. If, in the figure below, the area of the shaded sector is 85 % of the area of the entire circle, what is
the value of w?
(a) 15 (b) 30 (c) 45 (d) 54
(e) 60
7. The circumference of a circle is a units, and the area of the circle is b square units if a= b , what is
the radius of the circle?
(a) 1 (b) 2 (c) 3 (d)
(e)
Questions 8-9 refer to the following figure.
8. What is the length of the arc RS?
(a) 8 (b) 20 (c) 8 (d) 20
(e) 40
9. What is the area of the shaded sector?
(a) 8 (b) 20 (c) 8 (d) 20
(e) 40
box
10. In the figure above, what is the value of x?
box
11. If A is the area and C the circumference of a circle, which of the following is an expression for A in
term of C?

(a) (b) (c) 2C (d) 2C2

(e)
12. What is the area of a circle whose radius is the diagonal of a square whose area is 4?
(a) (b) 2 (c) (d)
(e)
Answer Key
1 d 2. a 3. b 4. c 5. 6.5

6. d 7. B 8. C 9. e 10. 54

11. A 12. d

SOLID GEOMETRY
1. The sum of the lengths of all the edges of a cube is 6 centimeters. What is the volume, in cubic
Centimeters, of the cube?

(a) (b) (c) (d) 1


(e) 8
2. What is the volume of a cube whose surface area is 150?

3. What is the surface area of a cube whose volume is 64?


(a) 16 (b) 64 (c) 96 (d) 128
(e) 384
4. What is the number of the cubic inches in one cubic foot?
(a) 12 (b) 24 (c) 144 (d) 684
(e) 1728
5. A solid metal cube of edge 3 feet is placed in a rectangular tank whose length, width, and height
are 3, 4 , and 5 feet, respectively. What is the volume, in cubic feet , of water that the tank can now
hold?
(a) 20 (b) 27 (c) 33 (d) 48
(e) 60
6. A 5- foot – long cylindrical pipe has an inner diameter of 6 feet and an outer diameter of 8 feet. If
the total surface area. (inside and out, including the ends) is k what is the value of k?
(a) 7 (b) 40 (c) 48 (d) 70
(e) 84
7. The height, b, of a cylinder is equal to the edge of a cube. If the cylinder and cube have the same
volume, what is the radius of the cylinder?

(a) (b) (c) (d)


(e)
8. A rectangular tank has a base that is 10 centimeters by 5 centimeters and a height of 20
centimeters. If the tank is half full of water, by how many centimeters will the water level rise if
325 cubic centimeters of water are poured into the tank?
(a) 3.25 (b) 6.5 (c) 16.25 (d) 32.5
(e) 65
9. If the height of a cylinder is 4 times its circumference, what is the volume of the cylinder in terms
of its circumference, C?

(a) (b) (c) (d)


(e)
10. Three identical balls fit snugly into a cylindrical can: the radius of the spheres equals the radius of
the can, and the balls just touch the bottom and the top of the can. If the formula for the volume of

a sphere is what fraction of the volume of the can is take up by the balls?

Answer key
1. a 2. 125 3 c 4. e 5. c
6. e 7. a 8. b 9. a 10 2/3
11. a 12 a 13 d 14. b 15 a

Analytical Reasoning
Stage 1
Question 1-3:
A chemist is preparing a nutrient using eight different vitamins and minerals … A, B, C, D, E, H,
F (Ferric), and Z (Zinc). According to the recipes, the following requirements apply to the use of
ingredients:
i. If B is used, both C and Z must also be used.
ii. E and H must always be used together.
iii. If C is used, at least two of A, B and F must also be used.
iv. C and H cannot be used together.
v. E, F and Z cannot all be used in a same nutrient.
vi. A, D and Z cannot all be used in the same nutrient.

Question 1:
1. Which of the following is a suitable combination of vitamins and minerals for a
nutrient?

a) A, B, C, F b) D, E, H, Z c) A, D, E, Z
d) C, D, E, F e) E, H, F, Z

2. Which of the following cannot be included in a nutriment that contains E?

a) B c) H e) Z
b) D d) F

3. By the addition of exactly one more mineral. Which of the following could make an
acceptable combination of vitamins and minerals?

a) A, D, Z c) C, D, H e) E, H, F
b) B, H, E d) C, E, Z

Question 4-5: At IOWA University, Students of Economics must complete a total of twelve
courses selected from three different parts of the syllabus…. Comparative economics,
environmental economics, and regional economics… in order to graduate, the students must
meet the following course distribution requirements:
i. At least six of the required twelve coursed must be from the environmental economics.
ii. At least five of the required twelve coursed must be from comparative economics and
regional economics, with at least one, but no more than three, selected from
comparative economics.
4. The minimum number of regional economics courses required to fulfill the course
distribution requirements is

a) 1 c) 5 e) 4
b) 3 d) 2

5. If the student has completed six environmental economics courses and one
regional economics course, the possible groups of courses to fulfill the course
distribution requirements must include at least:

a) One Environmental Economics d) Two Comparative Economics


Course Courses
b) Three Regional Economics Courses e) Three Comparative Economics
c) One Regional Economics Course Courses

Questions 6-8: Six candidates…… Ali, Amin, Omer, Hamza, Saleem and Osama are being
interviewed for a job. The interview will take place over four consecutive days, starting on
Thursday. Each candidate will have only one interview. The day on which the different
candidates will interview must confirm to the following conditions:
i. At least one interview will take place each day.
ii. No more than two interviews will take place on any day.
iii. No more than three interviews will take place on any two consecutive days.
iv. Ali’s interview must take place on Saturday.
v. Amin’s interview must take place on the same day with another interview.
vi. Saleem’s interview must take place on the day before Osam’s interview.
vii. Omer’s interview must take place on a day after Hamza’s interview.
6. If only one interview takes place on Thursday which candidate could have that
interview?

a) Ali c) Omer e) Osama


b) Amin d) Saleem
7. If the director decides to take two interviews on Thursday and two on Sunday,
how many candidates would be eligible to interview on Friday?

a) 1 c) 3 e) 5
b) 5 d) 4

8. If Hamza and Osama have their interviews on the same day which of the following
must be true?
a) Hamza’s interview will take place on Thursday
b) Saleem’s interview will take place on Friday
c) Amin’s interview will take place on Saturday
d) Osama’s interview will take place on Saturday
e) Amin’s interview will take place on Sunday

For Question 9 to 10
Three women ------ X, Y, and Z are traveling in a van with five children --- A, B, C, D and E. The
van has a driver’s seat and one passenger seat in the front, and two benches behind the front
seats, one bench behind the first, Each bench has room for exactly three people. Everyone
must sit in a seat or on a bench and seating is subject to the following restrictions: A women
must sit on each bench. Either X or Y must sit in the driver’s seat. C must sit immediately beside
E.
9. Which of the following can sit in the front passenger seat?

a) C c) X e) Z
b) D d) Y

10. Which of the following groups of three can sit together on a bench?

a) A,C and E c) A, Y and Z e) D, E and X


b) A, C and Z d) B, D and Y

11. If A sits immediately beside Z, which of the following CANNOT be true?

a) C sits immediately beside Y. d) A sits on the same bench as B.


b) D sits immediately beside Z. e) B sits on the same bench as X.
c) B sits in the front passenger seat.

12. If Y sits on a bench that is behind where C is sitting, which of the following must
be true?

a) B sits in a seat or on a bench that is c) A sits on the same bench as B.


in front of where E is sitting. d) D sits on the same bench as Y.
b) D sits in a seat or on a bench that is e) E sits on the same bench as Z.
in front of where A is sitting.

For Questions 13 to 15
Four computer operator (Ali, Babar, Cheema and Dar) each have to perform duties at het
NADRA on four different days, Thursday through Sunday. The following is their duty schedule:
Cheema has his duty day before Ali. Dar has his duty day later than Babar.
13. Which of the following is a possible order of duty days for the four operators?

a) Cheema, Dar, Ali and Babar b) Dar, Cheema, Ali and Babar
c) Babar, Cheema, Dar and Ali e) Ali, Babar, Dar and Cheema
d) Ali, Cheema, Dar and Babar

14. If Cheema has his duty day on Saturday, who must have his duty day on
Thursday?

a) Either Ali or Dar c) Ali e) Babar


b) Dar d) Either Babar or Dar

15. Each of the following possible EXCEPT:

a) Cheema has his duty on Thursday d) Babar has his duty on Sunday
b) Babar has his duty on Thursday e) Ali has his duty on Sunday
c) Dar has his duty on Saturday

For Question 16 to 20
The accounts staff of the Mark corporation presently consists of three book-keepers (X,Y and Z)
and five Data Entry Operators (M,N, O, P and Q). Management is planning to open a new office
in another city sending three Data entry Operators and two book- keepers from the present
staff. To do so they plan to separate certain individuals who do not function well together. The
following guidelines where established to set up the new office:
i. Book Keepers X and Z are constantly finding faults with one another therefore should
not be sent together to the new office.
ii. Z and N function well alone but not as a team. They should be separated.
iii. M and P have not been on speaking terms for many months. They should not go
together.
iv. Since M and O have been competing for a promotion, they should not be in one team.
Based on the above information, find the correct answers to the following questions:
16. If y is insists on staying back then how many combinations are possible?

a) 3 c) 1
b) 2 d) None

17. If X is to be moved as one of the book keepers, which of the following CANNOT be
a possible working unit?

a) XYMNQ c) XYMPQ
b) XYNOQ d) XYNPQ

18. If Z is sent to the new office then which member of the staff CANNOT be sent?

a) N c) O
b) Y d) P

19. If M is sent to the new office then which of the following is a possible team?

a) XYMNP c) YZMNQ
b) YZMOQ d) XYMNQ

20. If both Z and O are moved to he new office, how many combinations are possible?

a) 1 c) 2
b) 4 d) 3
Direction: For questions 21 to 24
Each of the following problems has a question and two statements which are labeled 1 and 2.
Use the data given in 1 and 2 together with other information given in the statement, and find a
correct answer by using basic mathematics and everyday facts.
21. How many bulbs does Munir have?
1. He bought two boxes each containing 12 bulbs.
2. He lent three bulbs to Khalid.
a) Statement 1. ALONE is sufficient but 2. ALONE is not sufficient to answer this question.
b) Statement 2. ALONE is sufficient but 1. ALONE is not sufficient to answer this question.
c) Statements 1 and 2. TOGETHER are sufficient to answer the question but NEITHER of
them is sufficient ALONE.
d) Statements 1 and 2. COMBINED are not sufficient to answer the question and additional
information is needed to find the correct answer.
22. If M > N and O > P, then M + O > N + P .is S> T?
1. S + A > T + B
2. A > B
a) Statement 1. ALONE is sufficient but 2. ALONE is not sufficient to answer this question.
b) Statements 2. ALONE is sufficient but 1. ALONE is not sufficient to answer this question.
c) Statements 1 and 2, COMBINED are sufficient to answer the question but NEITHER of
them is sufficient ALONE.
d) Statements 1 and 2. COMBINED are not sufficient to answer the question and additional
information is needed to find the correct answer.
23. In Lahore Zoo, the there are 37 deer. How many small black deer are there?
1. 12 of deer are small.
2. There are 20 black deer in the Zoo.
a) Statements 1. ALONE is sufficient but 2. ALONE is not sufficient to answer this question.
b) Statement 2. ALONE is sufficient but 1. ALONE is not sufficient to answer this question.
c) Statements 1 and 2. TOGETHER are sufficient to answer the question but NEITHER of
them is sufficient ALONE.
d) Statements 1 and 2. COMBINED are not sufficient to answer the question and additional
information is needed to find the correct answer.
24. Can there be more than 150 pictures in a 30-page book?
1. There is at least two pictures in each page.
2. There are no more than 4 pictures in any page.
a) Statement 1. ALONE is sufficient but 2. ALONE is not sufficient to answer this question.
b) Statement 2. ALONE is sufficient but 1. ALONE is not sufficient to answer this question.
c) Statements 1 and 2. TOGETHER are sufficient 1 and 2. TOGETHER are sufficient to
answer the question but NEITHER of them is sufficient ALONE.
d) Statements 1 and 2. COMBINED are not sufficient to answer the question and additional
information is needed to find the correct answer.

Questions 25 to 30 depend on the following passage


A Government Collage sports president wishes to select four members of a sports-wing
committee as special representatives to meet the requirements of college’s sports activities.
The committee consists of eight members four of which (K, L, M and N) are sports teaches
whereas the other four (P,Q, R and S) are students.
The president can select any four of the eight committee members as long as the following rules
are observed:
The four representatives must consist of exactly two sports teachers and two students.
Either K or L must be one of the representatives but K and L both cannot be the
representatives. If P is a representative then M must also be a representative.
If R is a representative the L cannot be a representative.
25. If R is a representative but M is not a representative then the whole group can be
determined if it were also true that:

a) K is a representative c) P is a representative
b) N is representative d) S is not a representative

26. If P is a representative then which of the following CANNOT be a representative.

a) M c) R
b) N d) Q

27. If L is a representative then which of the following can be the other three
representatives.

a) K, Q and S c) M, P and Q
b) M, N and P d) N, P and S

28. If neither Q nor S is a representative then which of the following is a pair of


teachers representatives?

a) K and L c) K and N
b) K and M d) L and M

29. If L, N and Q are representative then which of the following must also be
representative?

a) M c) R
b) P d) S

30. If K and N are representatives then which of the following is not a representative?

a) Q c) P
b) R d) None

31. What day of the week is today?


1. Today is December 25.
2. Amjad left Pakistan on Monday.
a) Statement 1. ALONE is sufficient but 2. ALONE is not sufficient to answer this
question.
b) Statement 2. ALONE is sufficient but 1. ALONE is not sufficient to answer this
question.
c) Statements 1 and 2. TOGETHER are sufficient to answer the question but NEITHER
of them is sufficient ALONE.
d) Statements 1 and 2. COMBINED are not sufficient to answer the question and
additional information is needed to find the correct answer.
32. Can any of the four rivers be more than 200 meters wide?
1. The narrowest of the four rivers is 140 meters wide.
2. Average width of the four rivers is 200 meters.
a) Statement 1. ALONE is sufficient but 2. ALONE is not sufficient to answer this
question.
b) Statement 2. ALONE is sufficient but 1. ALONE is not sufficient to answer this
question.
c) Statements 1 and 2. TOGETHER are sufficient to answer the question but NEITHER
of them is sufficient ALONE.
d) Statements 1 and 2. COMBINED are not sufficient to answer the question and
additional information is needed to find the correct answer.
33. If it is raining then there must be clouds. Are there clouds?
1. Today is Saturday. It is not raining.
2. It rained Friday.
a) Statement 1. ALONE is sufficient but 2. ALONE is not sufficient to answer this
question.
b) Statement 2. ALONE is sufficient but 1. ALONE is not sufficient to answer this
question.
c) Statements 1 and 2. TOGETHER are sufficient to answer the question but NEITHER
of them is sufficient ALONE.
d) Statements 1 and 2. COMBINED are not sufficient to answer the question and
additional information is needed to find the correct answer.

For questions 34 to 35
There are seven cages next to each other in a zoo. The following is known about the cages.
Each cage has only one animal, which is either a lion or a monkey. There is a lion in each of the
first and last cages. The cage in he middle has a monkey. No two adjacent cages have
monkeys in them. The monkey’s cage in the middle has two lion cages on either side. Each of
the other monkey cages are between and next to tow lion cages.
34. How many cages have lions in them?

a) 3 c) 4 e) 5
b) 2 d) 6

35. The monkey cage in the middle must have:

a) No other monkey cage to its left. d) Other monkey cages next to it.
b) No lion cage on its right. e) No lion cage to its left.
c) A lion age to its left and to its right.

Question 36 to 38
Seven Children --- M, N, O, P, Q, X and Y are eligible to enter a drawing contest. From these
seven, two teams must be formed, a blue team and a yellow team, each team consisting of
exactly three of the children. No child can be selected for more than one team. Team selection
is subject to the following restrictions: if P is on the yellow team. Q cannot be on the same team
as X. N cannot be on the same team as O.
36. Which of the following can be the three members of the blue team?

a) M, N and O c) N, O and Y e) P, Q and Y


b) M, Q and Y d) O, P and Q

37. If P and M are both on the blue team, , the yellow team can consist of which of the
following?
a) N. O and Q c) O, Q and X e) Q, X and Y
b) N, X and Y d) O, Q and Y

38. If P is on the blue team, which of the following if selected, must also be on the
blue team?

a) M c) Q e) Y
b) N d) X

Question 39 - 43
During 2006, from January though June, the Chairman of Physics Department will be on
Sabbath. The Dean of College has asked each of the college six professors in the department
---- Akhter, Bilal Chohan, Fraz, Hamid and Noman --- to serve as acting chairman during one of
these months. The physicists can decide the order in which they will serve, subject only to the
following criteria established by the dean.
i. Chohan will serve as chairman in February.
ii. Akhter will serve as chairman before Hamid does.
iii. Bilal and Fraz will serve as chairman in consecutive months.
39. Which of the following professors could serve as chairman in January?

a) Bilal c) Fraz e) Noman


b) Chohan d) Hamid

40. In how many ways can the schedule be made up if Noman has to serve as
chairman in may?

a) 1 c) 6 e) 2
b) 3 d) 4

41. If Noman serves in April, all of the following could be except:

a) Akhter serves in January d) Bilal serves in June


b) Hamid serves in march e) Hamid serves in June
c) Bilal Serves in May

42. If Bilal serves in May, what is the latest month in which Akhter could serve?

a) March c) January e) June


b) April d) February

43. Which of the following CANNOT be true?


a) Akhter and Noman serve in consecutive months
b) Noman and Hamid serve in consecutive months
c) Hamid and Fraz serve in consecutive months
d) AKhter and Chohan serve in consecutive months
e) Bilal and Chohan serve in consecutive months.

Question 44 – 48
During practice matches, before a major tournament, in a football ground, one team can
practice a time. There are seven teams --- the Argentine, the Brazil, the Senegal, the Dubai, the
England, the France and the Germany. The football ground is open seven evenings a week
from Monday to Sunday (Sunday being considered the last day of the week), and the allocation
of he practice times is governed by the following rules:
i. On any evening, only one team can play.
ii. The argentine must practice on Monday.
iii. The Dubai practice exactly one day before the France practice.
iv. The France practice exactly one day before the Germany practice.
v. The Senegal and the Brazil must practice earlier in the week than the England.
44. The latest day in the week that the Brazil can practice is:

a) Tuesday c) Thursday e) Saturday


b) Wednesday d) Friday

45. If a person went to the football ground on three consecutive evenings, her or she
could see which of the following teams in the order listed?
a) The France, the Germany, the Senegal.
b) The France, the Germany, the Dubai
c) The Argentine, the Dubai, the Senegal
d) The Brazil, the Senegal, the France
e) The Dubai, the England, the France
46. One week, the Senegal Practiced on Wednesday and the Dubai practiced the next
day. That week, the Brazil must have practiced on:

a) Monday c) Friday e) Sunday


b) Tuesday d) Saturday

47. If the Germany practice on Thursday, the England and the Dubai must practice on
which days, respectively?

a) Sunday and Tuesday d) Wednesday and Thursday


b) Saturday and Tuesday e) Tuesday and Monday
c) Friday and Wednesday

48. If the France practice on Saturday, the England must practice on what day?

a) Tuesday
b) Wednesday d) Friday
c) Thursday e) Sunday

ANSWERS ANALYTICAL STAGE# 1


1- B 2- A 3- E 4- D 5- C 6-D 7-D 8-E
9- B 10- D 11- E 12- E 13- C 14-E 15-D 16-D
17- C 18- A 19- D 20- A 21- C 22- C 23- D 24- B
25- D 26- B 27- C 28- B 29- D 30- C 31- D 32- C
33- D 34- C 35- C 36- E 37- D 38- B 39- E 40- E
41- E 42- A 43- A 44- E 45- B 46- B 47- A 48- C
Stage 2
Question Seven airline flights ----- 101,102,103,104,105,106 and 107 ---- are to be scheduled
for departure, one at a time on the hour, from 9:00 a.m until 3:00 p.m. The schedule must
conform to the following requirements:
 Flight 101 must depart at 9:00 a.m.
 Flight 105 must depart later than fight 103, and also later than Flight 102.
 Flights 104. 106 and 107 must depart on consecutive hours in that order.
1. If flight 107 is scheduled to depart at noon, Flight 105 must be scheduled to depart
at:

a) 10:00 a.m c) 1:00 p.m e) 3:00 p.m


b) 11:00 a.m d) 2:00 p.m

2. If flights 103 and 104 are scheduled to depart at 11:00 a.m. and 12 noon,
respectively, Flight 102 must be scheduled to depart at:

a) 9:00 a.m. c) 1:00 a.m e) 3:00 a.m


b) 10:00 a.m d) 2:00 a.m

3. Which of the following lists three flights in a sequence, from first to last, in which
they could be scheduled to depart consecutively?

a) 101,104,103 c) 104,105,106 e) 106,107,104


b) 102,103,106 d) 106,107,103

4. If Flight 106 is scheduled to depart at 2:00 p.m., Flight 105 must be scheduled to
depart at:

a) 10:00 a.m b) 11:00 a.m c) 12 noon


d) 1:00 p.m e) 2:00 p.m

5. Which of the following must be true about the scheduled order of the flights?
a) Flight 103 is scheduled to depart later than Flight 102
b) Flight 104 is scheduled to depart later than Flight 103
c) Flight 105 is scheduled to depart later than Flight 104
d) Flight 106 is scheduled to depart later than Flight 105
e) Flight 107 is scheduled to depart later than Flight 106
6. What is the latest hour at which Flight 102 can be scheduled to depart?

a) 10:00 a.m c) 12 noon e) 2:00 p.m


b) 11:00 a.m d) 1:00 p.m

Question: Exactly four young wood-carvers at to attend a wood-carving workshop. The four
are to be chosen from a group of seven eligible wood-carvers: three boys --- Frank, George and
Harlod ---- and four girls ---- Juanita, Karla, Mona, and Nellie. The four are to be chosen
according to the following conditions:
 If either George or Harold attends the workshop, the other must also attend.
 Juanita and Karla cannot both attend the workshop.
 George and Mona cannot both attend the workshop.
7. Which of the following could be the group attending the workshop?

a) Frank, George, Harold, Mona d) George, Juanita, Mona, Nellie


b) Frank, George, Karla, Nellie e) Juanita, Karla, Mona, Nellie
c) Frank, Karla, Mona, Nellie

8. If Frank and Juanita attend the workshop, which of the following pairs of wood-
carvers could be the others attending?

a) George and c) Harold and e) Mona and


Karla Mona Nellie
b) George and d) Karla and Mona
Nellie

9. If Harold attends the workshop, which of the following CANNOT attend?

a) Frank c) Juanita e) Nellie


b) George d) Mona

10. If two wood – carvers of each sex attend the workshops, which of the following is
one of the wood-carvers who must attend?

a) Frank
b) Harold
c) Juanita
d) Karla
e) Mona

ANSWERS ANALYTICAL STAGE # 2

1-E 2-B 3–D 4-C 5-E


6-E 7-C 8–E 9-D 10 - B
Stage 3
Question 1-3: From exactly seven objects – R, S, T, U, X, Y, and Z – a group of exactly four
objects must be selected in accordance with the following conditions:
 If R is selected, T must also be selected
 If S is selected, U must also be selected
 If X and Y are both selected together, T cannot be selected.
1. If X and Y are both selected together, which of the following must also be
selected?

A. R C. T E. Z
B. S D. U

2. If S and Z are both selected together, each of the following could also be selected.

A. R C. U E. Y
B. T D. X

3. If U is not selected, which of the following can be, but does not have to be
selected?

A. R C. T E. Z
B. S D. X

Question 4-9: A certain group of eight people is made up of two women – S and T; three
men – U,V, and W; and three children – X, Y and Z. Each of the eight must be on one, and only
one, of three teams being formed according to the following conditions.
 Each team must have either two or three members.
 Each team must have at least one child.
 S and X cannot be on the same team.
 V and W cannot be on the same team.
 U and Y must be on the same team.
4. If U on the same team as V, which of the following pairs must be on a team
together.

A. S and Y C. T and W E. W and X


B. S and Z D. T and Y

5. If is on the same team as Y, which of the following could be true?


A. U is on a team with only on other member.
B. T is on a team with only one other member.
C. T is on the same team as V.
D. W is on the same team as Y.
E. Z is on the same team as U.
6. If the two women are on a team together, which of the following must be on a
team, which only one other member?
A. U C. W E. Z
B. V D. X

7. If two of the men are on a team together, which of the following must be on a team
that has three members?

A. S C. V E. Y
B. T D. W

8. If S is on the same team as U, which of the following could be true?


A. The two women are on a team together
B. Two of the men are on a team together.
C. S is on a team with V
D. T Is on a team with Z
E. U is on a team with W.
9. If one man is on each team, any of the following pairs could be together on a team
EXCEPT:

A. S and T C. T and U E. V and Z


B. S and W D. T and W

Directions: Each question or group of questions is based on a passage or set of


conditions.
In answering some of the questions. It may be useful to draw a rough
diagram.
For each questions, selected the best answer choice given.

Question 10-13: Six products – U, V, W, X, Y and Z – are to be placed in the display window of
a vending machine with six compartments. Numbered 1 through 6 form left to right. The
products must be placed in the window, one product in each compartment, according to the
following conditions.
 U cannot be immediately to the left or immediately to the right of V.
 W must be immediately to the left of X.
 Z cannot be in compartment 6.
10. Which of the following products CANNOT be placed in compartment 1?

A. U C. W E. Z
B. V D. X

11. If X is placed in compartment 3, W must be placed in compartment .

A. 1 C. 4 E. 6
B. 2 D. 5
12. If U is placed in compartment 5, which of the following products must be placed in
compartment 6?

A. V C. X E. Z
B. W D. Y

13. If Z is placed in compartment 3, immediately to the right of X, which of the


following products must be placed in compartment 5?

A. U C. W E. Y
B. V D. X

Question 14-20:The manager of a commercial printing firm is scheduling exactly six jobs - P,
Q, S, T, W, and X – for a particular week. Monday through Saturday. Each job cab be
completed in one full day and exactly one job will be completed in one full day and exactly one
job will be scheduled for each day . the jobs must be scheduled according to the following
conditions.
 P must be printed sometime before S is printed.
 T must be printed don the day immediately before or the day immediately after the day
on which X is printed.
 W must be printed on Thursday.
14. Which of the following is and acceptable schedule of jobs for the week?

Mon Tues Wed Thurs Fri Sat

(A) P Q T W X S

(B) P W S X T Q

(C) Q X T W S P

(D) T X P W Q S

(E) X P T W S Q

15. Any of the following could be printed on Saturday EXCEPT:

A. P C. S E. X
B. Q D. T

16. If Q is printed on Wednesday, which of the following could be true?

A. P is printed on Tuesday D. T is printed on Monday


B. S is printed on Monday E. X is printed on Thursday
C. S is printed on Friday

17. If X is printed on Monday, which of the following must be true?

A. P is printed sometime before Q C. Q is printed sometime before S


B. P is printed sometime before W D. W is printed sometime before Q
E. W is printed sometime before S

18. If P is printed on Tuesday, which of the following must be true?

A. Q is printed on Monday D. T is printed on Friday


B. S is printed on Thursday E. X is printed on Saturday
C. S is printed on Saturday

19. If T is printed on Tuesday, any of the following could be true EXCEPT:

A. P is printed on Monday D. S is printed on Friday


B. Q is printed on Saturday E. X is printed on Wednesday
C. S is printed on Wednesday

20. If Q is printed on Friday, Which of the following must be true?

A. P is printed on Monday D. T is printed on Monday


B. P is printed on Wednesday E. X is printed on Tuesday
C. S is printed on Saturday

Question 21-25:A florist has exactly seven varieties of flowers ----- P, Q, R, S, T, U, and V ----
from which she must select combinations of exactly five varieties with which to make flower
arrangements. Any combination of the five varieties that conforms to all of the following
conditions is acceptable.
 If P is used in an arrangement, T cannot be used in that arrangement.
 If Q is used in an arrangement, U must also be used in that arrangement.
 If R used in an arrangement, T must also be used in that arrangement.
21. Which of the following is an acceptable combination of varieties that the florist
can select for and arrangement?

A. P,Q, S, T, U D. Q, R, S, U, V
B. P, Q, R, U ,V E. Q, R, S, T, U
C. P, S, T, U, V

22. If the florist selects variety R to be included in an arrangement, which of the


following must be true of that arrangement?

A. P is not used D. S is used


B. U is not used E. V is used.
C. Q is used

23. If variety P is used in an arrangement, which of the following CANNOT be sued in


that arrangement?

A. Q C. S E. V
B. R D. U
24. If the florist does not select variety V for an arrangement, which of the following
also CANNOT be selected?

A. P C. R E. T
B. Q D. S

25. Which of the following substitutions cab the florist always make without violating
the conditions governing flower combination, provided the variety mentioned first
was not, and the variety mentioned second was, originally going to be used in the
arrangement concerned.

A. P for R C. R for T E. V for T


B. Q for U D. S for V

Questions 26-31: Five ships --- J, K, L, M and N --- are to unloaded on 5 consecutive days
beginning on Monday and ending on Friday according to the following conditions:
 Each ship takes exactly one day to unload.
 K must be unloaded on a day preceding the days M and N are unloaded.
 L cannot be unloaded on Tuesday.
 M Must be the second ship unloaded after J is unloaded.
26. If M is unloaded on Friday, which of the following must be true?

A. J is unloaded on Wednesday D. L is unloaded on Thursday


B. K is unloaded on Tuesday E. N is unloaded on Thursday
C. L is unloaded on Monday

27. If K, M, and N are to be unloaded one immediately after the other in the that order,
the two days on which J can be unloaded are:

A. Monday and Tuesday D. Wednesday and Friday


B. Monday and Friday E. Thursday and Friday
C. Tuesday and Wednesday

28. If L in unloaded on the day immediately after the day J is unloaded, which of the
following must be true?

A. J is unloaded on Wednesday D. M is unloaded on Friday


B. K is unloaded on Monday E. N is unloaded on Tuesday
C. L is unloaded on Thursday

29. If J is unloaded on Monday, which of the following must be true?

A. L is unloaded on Tuesday D. L is unloaded on Thursday


B. L is unloaded before M E. N is unloaded on Thursday
C. K is unloaded on Tuesday
30. N cab be unloaded any day of the week EXCEPT:

A. Monday C. Wednesday E. Friday


B. Tuesday D. Thursday

31. On which of the following days can any one of the five ships be unloaded?

A. Monday C. Wednesday E. Friday


B. Tuesday D. Thursday

Questions 32-35: A contractor will build five houses in certain town on a street that currently
has no house on it. The contractor will select from seven different models of house ---- T, U, V,
W, X, Y, and Z the town’s planning board has placed the following restrictions on the contractor.
 No model can be selected for more that one house.
 Either model W must be selected to model Z must be selected, but both cannot be
selected.
 If model Y is selected, then model V must also be selected
 If model U is selected, then model W cannot be selected.
32. If model U is one of the models selected for the street, the n which of the following
models must also be selected?

A. T C. X E. Z
B. W D. Y

33. If T, U, and X are three of the models selected for the selected for the street, then
which of the following must be the other two models selected?

A. V and W C. V and Z E. Y and Z


B. V and Y D. W and Y

34. Which of the following is an acceptable combination of models that can be


selected for the street?

A. T, U, V, X, Y C. T, V, X, Y, Z E. V, W, X, Y, Z
B. T, R, X, Y, Z D. U, V, W, X, Y

35. If model Z is one model not selected for the street, then the other model NOT
selected must be which of the following?

A. T C. V E. X
B. U D. W

Questions 36-39: A group of three objects must be selected form six objects --- k, O, S, T, V
and W --- according to the following.

 K or S or both must be selected


 V or O must be selected
 Neither V nor S will selected with O
36. Which of the following is an acceptable selection of objects?

A. K, O, and S D. O, S and V
B. K, S and T E. O, T, and V
C. K, S, and V

37. Which of the following pairs of objects CANNOT both the amount the objects
selected?

A. K and O C. O and W E. V and W


B. K and T D. T and W

38. If S is selected, which of the following must also be among the objects selected?

A. K C. T E. W
B. O D. V

39. If V is not selected, which pair of objects must be among those selected?

A. K and O C. K and W E. O and W


B. K and T D. O and T

Questions 40-42: Exactly seven people --- Q, R, S, T, X, Y and Z ---- serve on an advisory
board. Q,R,S, and Z and have been elected to the board, and X, Y, and Z have been appointed
to the board. Three person or four person panels are sometimes drawn from he board to study
proposals. Each panel must include at last one elected and at least one appointed board
member, but no panel can consist of equal numbers of elected and appointed members. Each
panel is chaired by a person who is member of the group of board members (elected or
appointed) whose representatives are in the minority on that panel. Any panel must also
conform to the following conditions:
 If Q serves on a panel, T cannot serve on that panel.
 If R serves on a panel, X cannot serve on that panel.
 T and Y cannot serve on a panel unless they serve together.
 If Z serves on a panel, X must also serve on the panel.
40. Which of the following could be a panel drawn from the advisory board?

A. Q,R,S C. T,Y,Z E. R,T,X,Y


B. S,X,Z D. Q,S,T,Y

41. If R serves on a panel, it must be true that:

A. It is a three person panel D. T Chairs the panel


B. It is a four person panel E. Y chairs the panel
C. R chairs the panel

42. Each of the following could chair a panel EXCEPT:

A. S B. T C. X
D. Y E. Y

Question 43-48: A college has decided to appoint a four member committee to review
educational policy. Eight faculty members are eligible to serve on the committee: F, G, J , K, L,
N, O and P. three of these eight --- G, L, and N ---- are untenured, whereas the remaining five
are tenured, college policy and the personalities of the eight people in question dictate that the
choice of committer member meet the following conditions:
 The committee must be composed of exactly two untenured faculty members.
 G and J cannot serve together.
 If P is on the committee, then neither N nor O can be on the committee.
 If K is on the committee, then J must be on the committee,
 If both F and J are on the committee, then L cannot be on the committee.
43. If P is on the committee, the other three members of the committee must be:

A. F,G and L D. G, K, and L


B. F,G and N E. J, K, and L
C. G,J and L

44. If J is on the committee, which of the following is one of the faculty members who
must also be on the committee?

A. F C. L E. P
B. K D. O

45. If G is on the committee, any of the following could be on the committee EXCEPT:

A. K C. N E. P
B. L D. O

46. If neither F nor P is on the committee, which of the following could be list of the
committee members?

A. G, J, L, and O D. J, K, L, and O
B. G, K, N, and O E. J, L, N and O
C. G, L, O, and K

47. If L is not on the committee, which of the following is a list of people all of whom
must be on the committee?

A. F,G, and N D. J, K, and N


B. F,J, and N E. J, N and P
C. G, K, and N

48. If L and N do not both server on the committee, which of the following is a list of
people all of the whom could be on the committee together?

A. F,G, and L C. F,N, and P


B. F,J, and N D. G,K and N
E. K,L and P

Question 49-55: Six puppies --- K, L, M, S, T and U --- must each be scheduled for examination
by a veterinarian. The puppies are to be examined on at time in six consecutive time slots on
the same day according to the following conditions:
 M Cannot be examined immediately before or immediately after S.
 L must be examined immediately before U.
 K must be examined fourth.
49. Which of the following is an acceptable examination schedule for the puppies, in
order from first examined last examined:

A. L, T, S, K, M, U D. S, T, M, K, L, U
B. L, U, T, K, S , M E. T, M, S, K, L, U
C. M, T, S, L, U, K

50. If L is examined second which of the following must be true?

A. K is examined at some time C. T is examined at some time after


before S K
B. M is examined at some time D. M is examined sixth
before T E. S is examined first

51. S can be examined in any of the following time slots EXCEPT:

A. First C. Third E. Sixth


B. Second D. Fifth

52. If L is examined first, T must be examined:

A. Immediately before K D. At some time after K


B. Immediately before S E. At some time after S
C. At some time before M

53. If S examined sixth, which of the following is a complete and accurate list of the
time slots any one of which could be the time slot in which M is examined?

A. First D. First, second, third


B. First, second E. Second, third, fifth
C. First, third

54. If U is examined at some time before M is examined , L can be examined.

A. Immediately after S D. At some time after M


B. Immediately after T E. At some time after K
C. Immediately before T

55. If both M and T are examined at some time after K is examined, S must be
examined:
A. First D. First or else third
B. Second E. Second or else third
C. Third

Question 56-62: At an art show, exactly five sculptures are to be displayed in a room with
exactly five stands, arranged along a single wall and numbered consecutively I through 5. The
sculptures are to be selected from a total of eight sculptures ---- M, N, P, Q, R, S, T, and U ----
and displayed one sculpture on each stand, according to the following conditions:
 Either M or U both or must be selected.
 I f M is selected, M must be on stand 1.
 Either R or S must be on stand 3.
 If T is selected , P must also be selected, and T and P must then be on stands are
immediately adjacent to one another.
56. Which of the following is an acceptable selection of sculptures to be displayed on
stands I through 5?

1 2 3 4 5
(A) M R T P Q
(B) N T S U Q
(C) P T S R U
(D) T P R S M
(E) U N Q P T
57. If S is on stand 1, which of the following must be true?

A. P is on stand 4 D. T is on stand 4
B. R is on stand 3 E. U is on stand 5
C. T is on stand 2

58. If T is on stand 5, which of the following pairs of sculptures can be on stands that
are immediately adjacent to each other?

A. M and P C. Q and P E. U and R


B. Q and N D. R and T

59. If U is on stand 4, any of the following can be on stand 5 EXCEPT:

A. N C. Q E. T
B. P D. R

60. If T is on stand 2, which of the following sculptures must be selected?

A. M C. R E. U
B. N D. S

61. If P is not selected and R is on stand 1, which of the following lists, in alphabetical
order, those sculpture’s that must also be selected?

A. M,Q, T, and U B. N, Q, S, and T


C. N, Q, S, and U E. Q, S, T, and U
D. N, S, T, and U

62. If Q is displayed on a stand immediately adjacent to a stand on which R is


displayed and immediately adjacent to a stand on which S is displayed, which of
the following must be true?

A. N is on either stand 4 or stand 5 D. S is on either stand 3 or stand 5


B. Q is on either stand 2 or stand 4 E. U is on either stand 2 or stand 4
C. R is on either stand 1 or stand 3

Question 63-66: Seven meetings --- J, K, L, M, N, O and P ---- are to be schedules, one on
each day of a week that begins on Sunday.
 Meeting J must take place on Sunday.
 Meeting K must take place after both meeting L and Meeting M.
 Meeting N, O and P must take place on three consecutive days, not necessarily in that
order.
63. Which is the latest day of the week on which meeting L can take place?

A. Tuesday C. Thursday E. Saturday


B. Wednesday D. Friday

64. Which of the following must be true about the order of meeting?

A. L takes place after J D. P takes place after P


B. L takes place after O E. takes place after P
C. N takes place after O

65. If meeting O is on Saturday, then meeting K must take place on:

A. Monday C. Wednesday E. Friday


B. Tuesday D. Thursday

66. Which of the following represents a possible ordering of meetings on three


consecutive days?

A. JMK C. MNJ E. POM


B. KLO D. OJN

Question 67-71:Seven dignitaries --- F, G, H, I, N, O and P ---- are to be seated together at a


diplomatic ceremony. They will be seated in a row of seven chairs, numbered form 1 to 7, form
front to back. Any seating is acceptable as long as all seven dignitaries are seated, one in each
chair, and the seating conforms to the following rules:
 F must sit in the chair immediately O’s chair.
 G cannot sit in the chair immediately in form of N’s Chair, and G cannot sit in the chair
immediately behind N’s Chair.
 There must be exactly two chairs between the chairs of H and P.
 There must be at least one chair between the chairs of I and P.
 N must sit in chair 3.
67. Which of the following seating arrangements, form chair 1 through chair 7,
conforms to the rules?

A. F, I, N, P, G, O, H D. I, H, N, P, O, F, G
B. G, P, N, I, H, O, F E. O, F, H, N, I, P, G
C. I, G, N, P, O, F, H

68. If F sits in chair 6 and H sits in chair 7, which of the following dignitaries must sit
in chair 2?

A. G C. N E. P
B. I D. O

69. If the seating arrangement, from chair 1 through chair 7, is G, I, N, H, O, F, P,


which of the following Paris of dignitaries can exchange seats without violating
the rules?

A. F and G C. G and I E. I and P


B. G and H D. H and P

70. If O sits in chair 1 and H sits in chair 7, then the number of chairs between F’s
chair and I’s chair must be:

A. Zero C. Two E. Four


B. One D. Three

71. If H sits chair 4 and F sits in chair 6, then the dignitaries in chairs 1 and 7,
respectively, must be:

A. G and O
B. G and P
C. I and P
D. O and I
E. P and O
1-D 19-C 37-D 55-D

2-A 20-C 38-D 56-C

3-D 21-E 39-A 57-B

4-B 22-A 40-B 58-E

5-B 23-B 41-E 59-E

6-D 24-A 42-E 60-E

7-E 25-D 43-A 61-C

8-E 26-A 44-C 62-B

9-A 27-A 45-A 63-D

10-D 28-B 46-E 64-A

11-B 29-C 47-A 65-C

12-D 30-A 48-A 66-E

13-E 31-C 49-D 67-B

14-D 32-E 50-C 68-B

15-A 33-C 51-B 69-D

16-D 34-C 52-D 70-D

17-E 35-B 53-C 71-B

18-A 36-C 54-A


ANSWER

Stage 4
Indian Analytical Reasoning
Various Types and Solved Examples
There the variety of problems under analytical Reasoning. Broadly, they can be categorized
under the following headings
1. Seating Arrangements
a) In a row
b) around a table
c) Circular
d) Any other shape (square, rectangular etc)
2. Sequencing
3. Combinations
4. Comparisons
5. Selections
6. Series-based
7. Ranking

Let us discuss and understand the details involved under each of these categories
1. SEATING ARRANGEMENT:

In these kinds of problems, some people are sitting in a row or around a table in a desired
formation. The conditions provide clues towards the actual arrangement and you have to make
use of these clues to reach to the final arrangement.
(a) Seating arrangement in a row: Let us understand the type with the help of the
following example.
Directions for questions 1 to 5: Answer the questions based on the following information.
I. A,B,C,D,E,F and G are sitting on a bench and all of them are facing East.
II. C is to the immediate right of D, but not next to F.
III. B is at the extreme end and has as his neighbor.
IV. G is between E and F.
V. D is sitting third from the South end .

1. Who is sitting to the right of E?


A. A C. D
B. C D. None

2. Which of the following pairs is sitting at the extreme ends?


A. A, B
B. A, E
C. C, B
D. F, B
3. The person sitting third form the North end is ___.
A. E
B. F
C. G
D. D

4. Between which of the following pairs is D Sitting?


A. A, C
B. A, F
C. C, E
D. C, F

5. Which of the conditions form I to v given above is not required to find out the
place where A is sitting?
I
Ii
Iii
All are required.
Solution for questions 1 to 5:
From (i): A, B, C, D, E, F, G are sitting on a bench and all of them are facing East.
…………………………………………………………………East
From (ii): DC
From (iii): BE………………………
OR
………………………………………………EB
From (IV): E G forFG e
For (V): ……………………………D…………………………. South
Let us start with the arrangement obtained form condition (v).
1234567T
Now, from (ii), we get that C will occupy seat 6. From (iii). B and E will occupy seats 1 and 2,
respectively. From (iv), G and F will occupy 3 and 4 and finally the last seat 7 will be occupied
by the remaining person A. From the above reasoning, we get the following final arrangement.

B E G F D C A East

1.4 G is sitting to the right of E.


2.1 A and B are sitting at the extreme ends.
3.3 G is sitting 3, d form the North end.
4.4 D is sitting between C and F.
5.4 All are required.
(b) Seating arrangement around a table: Let us consider the following example for circular
arrangements.
Directions for questions 6 to 10: Read the given information carefully and answer the
questions given below:
Six persons A, B, C, D, E and F are sitting around a circular table facing the center.
I. C is sitting exactly between A and F.
II. B is sitting two places to the left of E.
III. D is sitting two places to the right of F.
6. Between which two persons is D sitting?
A. F-B
B. E-B
C. C-B
D. A-B
7. Who is sitting opposite A?
A. F
B. C
C. E
D. None of these
8. Which of the following is A’s neighbor to his right?
A. C
B. F
C. B
D. D
9. Who is sitting opposite E?
A. A
B. B
C. C
D. F
10. Between which of the two persons is F sitting?
A. C–D
B. C–A
C. D–A
D. C–B

Solutions for questions 6 to 10:


Start with any fixed position. Statement I does not give any fixed position since the order could
be A-C-F or F-C-A. Starting with ii, we will have the positions of and E. Now, C has to be in
between A and F in such an order that D is two places to the right of F. The order in the
clockwise direction has to be F-C-A, else A will fall 2 places to the right of F.

Thus, we have the arrangement as shown below.


6.2 D is sitting between E and B
7.4 B is sitting opposite A.
8.1 C is to the immediate right of A.
9.4 F is sitting opposite E.
10.4 F is sitting between C and B.

2. SEQUENCING:
In such type of problems, certain things or events have to be arranged in a
sequence or an order as per the conditions. Let us look at the following example
for better understanding.
Directions for questions 11 to 14: Read the following information, carefully and answer the
questions given below.
I. Seven meetings – A, B, C, D, E, F and G are to be scheduled, one on each day of a
week that begins on Monday.
II. Meeting A must take place on Monday and meeting B on the last day.
III. Meeting B immediately takes place after meeting C which is scheduled immediately after
meeting D.
IV. Meeting E, F and G must take place on three consecutive days, in that order.
11. Which is the earliest day of the week on which meeting C can take place?

Wednesday
Thursday
Friday
Saturday
12. Which of the following must be true about the order of meetings?
A. C takes place immediately after A.
B. C takes place immediately after F.
C. E takes place immediately after A
D. E takes place immediately after G.
13. If meeting A is on Wednesday, which is the first day that meeting B must take
place on?
A. Tuesday
B. Wednesday
C. Thursday
D. Friday
14. Which of the following represents a possible order of meetings on three
consecutive days?
A. ADB
B. BCF
C. DEA
D. AEF

Solutions:
The given information can be summarized as follows:
Days Meetings
1- Monday --- A…. from statements (i) and (ii)
2- Tuesday --- E
3- Wednesday--- F…. from the statement (iv)
4- Thursday--- G
5- Friday ---- D….. from statement (iii)
6- Saturday ---- C
7- Sunday --- B… from statements (i) and (iii)
11.4 ----- Saturday
12.3 ------ E takes place immediately after A.
13.1----- from statement (ii), we know that meeting A takes place on Monday i.e., the first day,
and B takes place on the last day i.e., Sunday, if the first day changes from Monday to
Wednesday, then the last day becomes Tuesday.
14.4 ----- AEF, as can be observed form the arrangement.
Directions for questions 15 and 16: These questions are based on the following information.
Five friends ---Hemant, Ram, Krishna, Pramod and lahesh participated in race. Ram finished
the race before Krishna but after Hemant. Hemant finished the race before Mahesh and
Pramod. Pramodnished the race after Krishna but before Mehesh.
15. Who finished the race in the fourth position?
A. Krishna
B. Mahesh
C. Pramod
D. Ram
16. Who was the first persons to finish the race?
A. Hemant
B. Pramod
C. Ram
D. Mahesh

Solution:
Ram finished the race before Krishna but after Hemant who finished the race before Mahesh
and Pramod means Hemant must finished the race first. Pramod finished the race after Krishna
but before Mahesh.
So, the order we get in ranks is as follows.

Hemant Ram Krishna Pramod Mahes


1 2 3 4 5
15.3 Pramod finished the race in the fourth position.
16.1 Hemant finished the race first.
3. COMBINATIONS:
Here, the elements in some groups are to be combined, as per the given conditions. In the
following example, the groups are of (a) Men, (b) Professions and (c) Musical Instruments. As
per the conditions, these are mixed and matched.
Let us look at the following example.
Directions for questions 17 to 21: Read the following information carefully and answer the
questions given below:
i. Five gentlemen (Mr. Ajay, Mr, Bijay, Mr, Vinay, Mr Sanjay and Mr, Akshay) are
practicing five differnet professions (Engineering, Medical, Law, Chartered
Accountancy and Architecture). Each one can play only one of the five different
instruments; Table, Violin, Sarod, Sitar and Flute.
ii. Mr Ajay is a Doctor and can play Sarod.
iii. The Sitarist is not an Engineer.
iv. MrVinay and MrBijay are not Architects and Vinay cannot play Tabla.
v. MrBijay can play Violin.
vi. MrAkshay is a Lawyer and can play Flute.

17. Which instrument does Mr. Vinay play?


A. Sarod
B. Sitar
C. Violin
D. Flute

18. What is the profession of Mr. Bijay?


A. Architect
B. Doctor
C. Lawyer
D. Engineer

19. Who is an Architect?


A. Mr. Ajay
B. Mr. Akshay
C. Mr. Bijay
D. Mr. Sanjay

20. What is the profession of Mr. Vinay?


A. Doctor
B. Engineer
C. Lawyer
D. CA

21. Which instrument can the Doctor learn from the Architect?
A. Flute
B. Sitar
C. Tabla
D. Sarod

Solution:
Let us represent the three groups in a table. By taking the group of gentlemen as the base,
because most of the information given is with regard the gentlemen, we will try filling in the
other details/ elements of the other two groups in the table, as shown below.
Form ii, we get the combination Ajay-Doctor-Sarod.
From iii, we get to know that Sitar ≠ Architects and Vinay ≠ Tabla.
From iv, (Vinay, Bijay) ≠ Architects and Vinay ≠ Tabla.
From v, Bijay = Violin.
Form vi, we get the combination Akshay – Lawyer – Flute.
Putting the above details in the table as shown below.

Gentleme Professional Instrument


n
Ajay Doctor Sarod
Bijay X Architech Violin
Vinay X Architech X Tabla
Sanjay
Akshay Lawyer Flute
Now, here we observe that neither Bijay nor Vinay is the Architect; hence the remaining
person Sanjay is the Architect. Similarly, Sanjay plays Table and hence Vinay plays Sitar.
This means that Bijay is the Engineer (from iii) and Vinay is the CA. We get the final
arrangement as shown below:

Gentlema Professional Instrument


n
Ajay Doctor Sarod
Bijay Engineer Violin
Vinay CA Sitar
Sanjay Architect Tabla
Akshay Lawyer Flute
Now, based on the above table, let us answer the questions.
17.2 Mr. Vinay plays Sitar.
18.4 Mr. Bijay is the Engineer.
19.4 Mr. Ajay is the Architect.
20.4 Mr. Vinay is the CA.
21.3 The Doctor can learn Table form in Architect
4. Comparisons:
In such kind of problems, some elements are compared with each other in terms of
measurable (like height, weight, speed, size marks, etc.) Let us have a look at the following
example.
22. Among five boys, Vasant is taller the Manohar, But not as tall as Raju. Jayant I taller
than Dutta, But shorter than Manoha who is the tallest in the group?
A. Raju
B. Manohar
C. Vasant
D. Can’t be determine

Solution:
Arranging the given information, we get Raj>Vasant>Manohar>Jayant>Dutta so, Raju is the
tallest.
5. Selections:
In these problems, some teams are made form the given people in accordance with the
conditions. The most common statement are: 1. A and B are in the same team. 2. And B
cannot be in the same team.3. A and B are in different teams. Let us look at following
example.
23. Two teams of three members each have to be selected form among six persons – P, Q,
R, S, T and U, P and R cannot be in the same team. Q and S must be in the same team.
R and T cannot be in the same team. Which of the following must be one of the two
teams selected?
A. P, T and U
B. P, Q and T
C. P, S and R
D. Q, R and T

Solution:
As P and R can not be in the same team and R and T cannot be in the same team, R must
be with Q and S. Hence, the other team is P, T and U.
6. Series-Based
In these kind of problems, you’ll see a series consisting of numbers, letters or symbols
as the elements. Any one of these is selected and the following type of questions is
posed.
I. How many X and are such that each is immediately preceded by Y but not
immediately followed with Z?
II. How many X and are such that each is immediately preceded by Y but not
immediately followed with Z?
III. How many X and are such that each is immediately preceded by Y but immediately
followed with Z?
IV. How many X are such that each is neither immediately preceded by Y nor
immediately followed with Z?

Let us look at the following example.


24. How many 6’s are there in the following series of numbers which are preceded by 7 but
not immediately followed by 9.
A. One
B. Two
C. Three
D. Four

Solution
All the 6’s that satisfy the given condition are underlined in the series
6795697687678694677695ZJ3 so, in above series, 3 times, 6’s are preceded by 7 but
not immediately followed by 9.
7. Ranking:
Here, a student may have a rank from the top or bottom of the result ranking list. The
following generalization can be used solving such questions.
T = RT + RB-1
Where
T Total number of student in the class.
RT Rank from the top of the ranking list.
RBRanking from the bottom of the ranking list.
Let us look at the following example for better clarity.
25. In a class, Krishna is ranked 8 th from the bottom. How many students are there in his
class?
A.56
B.55
C.57
D.None of these

Solution:2
Using the formula T = RT + RB -1, we get T = 8 + 48 – 1=55. Hence, there are 55
students in Krishna ‘s class.
Now , please solve questions in the exercise based on the concepts discussed.
Exercise
Directions of questions 1 to 5: Answer the questions on the following information.
I. There are five friends.
II. They are standing in a row facing north.
III. Jayesh is to the immediate right of alok.
IV. Pramod is exactly between bhagat and subodh .
V. Subodh is exactly between jayesh and pramod .
1. Who is at the extreme left end ?
A. Alok
B. Bhagat
C. subodh
D. jsyesh
2. Who is in the middle
A.Bhagat
B.Jayesh
C.Pramod
D.Subodh
3. To find the answer to the above two questions, which of the given statement can
be dispenser with?
A.None
B.Only
C.Only
D.Only
4. If five of them were to stand in a circle with the same arrangement, between which
two people would Bhagat stand?
A.Alokand and Subodh
B.Jayesh and Pramod
C.Subodh and Pramod
D.Alok and Pramod
5. If a new friend Sukhdev joins the group, and is standing to the right of Bhagat,
who is his other neighbor (in the orginal linear arrangement)?
A.Jayesh
B.Pramod
C.Subodh
D.None

Directions for questions 6 to 10: Study the following information carefully and answer the
questions given below it.
Eleven students A, B, C, D, E, F, G, H, I, J and K are sitting in the first row of the class
facing the teacher.
D, who is to the immediate left of F, is second to the right of C.
A is second to the right of E, who is at one of the ends.
J is the immediate neighbor of A and B and third to the left of G.
J is second to the left of I.
6. Who is sitting in the middle of the row?
A.C
B.I
C.B
D.G
7. Which of the following group of friends could be sitting to the right of G?
A. IBJA
B. ICHDF
C. CHDF
D. CKDE
8. in the above seating arrangement, which of the following statements is
superfluous?
A.I
B.H
C.111
D.None of Superfluous

9. Which of the following statements is TRUE in the context of the above seating
arrangement?
A.There are three students sitting between D and G.
B.G and C are neighbors sitting to the immediate right of H.
C.B is sitting between J and I.
D.K is between A and J
10. If E and D, C and B, A and H and K and interchange their positions, which of the
following pairs of students is sitting at he ends?
A.D and E
B.E and F
C.D and K
D.K and F

Directions for questions 11 to 15: Read the giver information carefully and answer the
questions give below:
Eight persons L, M, N, P, Q, R, S and T are sitting for a round table conference facing the
centere.
I. R sits between L and S.
II. S, who is the neighbor of Q, sits 3 places to the right of T.
III. Q sits 2 palaces to the right of T.
IV. M sits 3 places to the left of R.

11. Who sits opposite M?


A. P
B. L
C. 0
D. T

12. Between which two persons is S sitting?


A. L-Q
B. M-Q
C. R-Q
D. L-M

13. Who sits opposite S?


A. N
B. P
C. T Either N or P

14. Who among the following is Q’s neighbor?


A. P
B. R
C. L
D. S

15. Who is L’s neighbor on his left?


A. R
B. S
C. 0
D. T

Directions for questions 16 to 20: Answer the questions based on the following information.
B, C, D, E, F and G are to be seated at a round able. The following apply to the seating
arrangement.
D must sit next to F.
B cannot sit next to F.
C cannot sit next to G.
16. If D is one of the two people who sit next to E, then which of the following can
sit next to E?
A. B
B. C
C. G
D. Either C or G

17. Who must sit on the chairs on either sit of E, if B sits next to D and C sits next to
F?
A. B and G
B. B and C
C. B and F
D. C and G

18. Who must sit directly across the table form F, what is the total number of seating
arrangements possible?
A. 1
B. 2
C. 3
D. 4

19. Who must sit in the chairs on either side of G, if C sits directly across the table
form E?
A. C arid D
B. D and E
C. E and F
D. B and E

Directions for questions 21 to 25: Read the information given carefully and answer the
questions that follow.
Eight persons L, M, N, P, Q, R, S and T are sitting around a square table such that there are
two on each side and they are all facing the center the table.
i. P sits between L and S.
ii. Q sits two places to the left of L.
iii. R and T are sitting along one side of the square table. R sits opposite L.
iv. M sits two places to the left of R.

21. Who sits opposite P?


A.S
B.M
C.N
D.T

22. Who sits two places to the right of S?


A.P
B.M
C.T
D.L

23. Between which two persons is L sitting?


A.M-P
B.N-P
C.N-R
D.T-Q

24. Which of the following is a neighbor of L?


A.S
B.Q
C.P
D.R

25. Who sits opposite Q?


A.S
B.P
C.T
D.M

Directions for questions 26 to 30: Read the given information carefully and answer the
questions given below:
Five books A, B, C, D and E have to be proofread is, 6 hours where one hour needs to be
spent per book.
i. A break of one hour has to be taken in the third or the fourth hour.
ii. The proofreading cannot start with A and has to end in C.
iii. D has to immediately follow B with no break In-between.
iv. A cannot be done immediately after D.
v. A has to immediately proceed E with no break in-between…

26. Which is the first book to be proofread?


A. Sixth
B. Fourth
C. Fifth
D. Third

27. Which is the first book to be proofread?


A. D
B. A
C. B
D. C

28. Which book is to be proofread immediately after the break?


A. D
B. B
C. A
D. C

29. Which book is to be proofread immediately after D?


A. B
B. E
C. C
D. None

30. Which book is to be proofread immediately after E?


A. A
B. E
C. C
D. B

Directions for questions 31 to 35: Read the given information carefully and answer the
questions given below:
Six lectures on 6 different subjects Physics, Chemistry, Biology, Algebra, Geometry and
Astronomy have to be scheduled (one on each day) across 7 days starting Sunday and ending
Saturday. The scheduler has to be drawn out for the subjects such that.
i. One day has to be a holiday and it can be neither Sunday nor Saturday.
ii. Geometry has to be scheduled immediately after Algebra.
iii. Physics cannot start the series in the week and has to be done exactly 2 days before
Astronomy.
iv. Biology has to be scheduled for Thursday and cannot immediately follow Physics.
31. What subject will start the series of lectures?
A. Algebra
B. Chemistry
C. Physics
D. Biology

32. Which of the following days is a holiday?


A. Monday
B. Tuesday
C. Wednesday
D. Thursday

33. On which day is the lecture in Physics scheduled?


A. Monday
B. Tuesday
C. Wednesday
D. Friday

34. On which day is the lecture in geometry scheduled?


A. Monday
B. Tuesday
C. Wednesday
D. Saturday

35. How many days after Physics is Biology scheduled?


A. One
B. Three
C. Four
D. Two

Directions for questions 36 to 40: Read if following information carefully and answer the
questions given below.
i. There are seven teachers ‘A’, ‘B’, ‘C\ ‘D\ ‘E ‘F’ and ‘G’ in a college. Each one of them
teachers a different subject.
ii. There are three female and four ma teachers, and out of these, there are two pairs of
couples.
iii. ‘C’ who teachers Social Sciences is married to the teacher who teachers Chemistry.
iv. ‘E’ and ‘G’ are female teachers who tea © Zoology and Physics respectively.
v. ‘A’ teaches Mathematics and his wife doe not teach Physics.
vi. ‘B’ does not teach Chemistry or Commerce
vii. ‘F’ and ‘D’ are male teachers. ‘F’ is unmarried

36. Which subject does ‘F teach?


A. Mathematics
B. Chemistry
C. Commerce
D. Social Science

37. Which subject does ‘B’ teach?


A. Physics
B. Commerce
C. Social Sciences
D. Cannot be determined

38. Which of the following are two pairs couples?


A. DC and AE
B. AC and DE
C. GA and CD
D. Cannot be determined

39. Which subject does A’s wife teach?


A. Chemistry
B. Zoology
C. Social Sciences
D. Cannot be determined

40. Who among the following are the males among the two couples?
A. AC
B. AE
C. AD
D. cannot be determined.

Directions for questions 41 to 45: Read the information given carefully and answer the
questions as follow.
Amit, Bharati, Cheryl, Deepak and Eric are five fiends sitting in a restaurant. They are wearing
caps {five different colours – yellow, blue, green, white and red. Also, they are eating five
different snacks burgers, sandwiches, ice-cream, pastries and pizza.
I. The person wearing a red cap is eating pastries.
II. Amit does not eat ice-cream and Cheryl is eating sandwiches.
III. Bharati is wearing a yellow cap and Amit is wearing a blue cap.
IV. Eric is eating pizza and is not wearing a green cap.

41. What is Amit eating?


A. Burgers
B. Sandwiches
C. Ice cream
D. Pastries

42. Who is wearing the green cap?


A. Amit
B. Bharati
C. Cheryl
D. Deepak

43. Who is eating ice-cream?


A. Amit
B. Bharati
C. Cheryl
D. Deepak

44. Which colour cap is Eric wearing?


A. Yellow
B. Blue
C. Green
D. White

45. Which of the following combinations is not corrects?


A. Yellow cap + ice crea
B. Red cap + pastires
C. White cap + pizza
D. Bharati + burger

Answer Key

1 1 2 4 3 1 4 4 5 4 6 2 7 3 8 4 9 3 1 3

1 2 1 3 1 4 1 4 1 1 1 4 1 4 1 1 1 4 2 4

2 4 2 3 2 2 2 3 2 4 2 4 2 3 2 2 2 4 3 3

3 2 3 2 3 1 3 4 3 2 3 3 3 4 3 1 3 2 4 3

4 1 4 3 4 2 4 4 4 4

Analytical Reasoning
For Questions 1 to 5:
From (i) and (ii):
-------------------- North
Form (iii):
Alok Jeyesh
Form (iv): (a) BhagatPramodSobodh OR (b) SoubodhPramodBhagat
Form (v): (a) JayeshSubodhPramod OR (b) PramodSubodhJayesh
Let us start with the arrangement obtained from condition (iii). AlokJayesh
Hence, (v) (a) Can’t be possible
From the above reasoning, we get the following final arrangement.
1.1 Alok is at the extreme left end.

2.4 Subodh is in the middle


3.1 All statements are necessary
4.4 Bhagat stands between Alok and Pramod.
5.4 As Sukhdev stands the extreme right end, he has only one 21 eithbor, Bhagat,
For questions 6 to 10:
----------------
1 2 3 4 5 6 7 8 9 10
From (ii):
C __ D F
From (iii): E/1 2 A/1
From (iv) (v): (a) A J B I G O R (b) B J A I G
Let us start with the arrangement obtained from condition (iii).
E / 1 2 A/ 1
Hence, (v) (b) Can’t be possible.
Form (iii), (iv) and (v) (a), we get the following arrangement.
Now, from (i), we get that C will occupy seat 8. Hence, we will get the following arrangement.

- - - - - - -- - - - -------- - -- - - -

So, H can occupy either seat 2 or seat 9. Also, K can occupy seat 2 or seat 9.
6.2 I is sitting in the middle of the row.
7.3 the group of friends sitting to the right of G Could be CHDF.
8.4 None is superfluous.
9.3 B is sitting between J and I.
10.3 the new arrangement is
DFJHJBAEK
So, D and K are sitting at two ends.
For questions 11 to 15:
S sits 3 places to the right of T and Q, 2 places to the right of T. Fix up these positions first.

Now, if R has to sit between L and S, it has to be to the right of S else Q will clash with R. We
can also get the position of M relative to R. However, the positions of N and P cannot be
determined for sure.

11.2 L sits opposite M.


12.3 S is sitting between R and Q.
13.4 Either Nor P sits opposite S..
14.4 S is Q’s neighbor.
15.1 R is to the left of L.
16.4 there are two arrangements possible.

Hence, ether C or G sites next to E.


17.4

Hence, C and G sit next to E.

18.1
B sits opposite F.
19.4 There are arrangements possible.

20.4 There are 2 arrangements possible.

In both the arrangements position G is between B and E only.


For questions 21. To 25:
Start by fixing the position of one of the persons. The best statement to start with is (iii), since
the two opposite positions are fixed simultaneously and the remaining positions can be derived
relative to these positions. There are two possible arrangements.
Using statement (ii), we get

Now, according to statement (iv) M sits 2 laces to the left of R which is not possible in
arrangement 1 as P is already present three. So, only arrangement 2 is possible. The final
arrangement is as follows:

21.4 T sits opposite P.


22.3 T sits two places to the right of S.
23.2 L is sitting between N and P.
24.3 P is L’s neighbor.
25.4 M sits opposite Q.
For questions 26 to 30:
C is the last book. The combinations B-D and A-E in that order have to compulsorily exist. Now,
the order will have to start with B-D, since A cannot be started with (from (ii)).
1 2 3 4 5 6
B D C

If the fourth hour is the break, then the combination A-E cannot be fitted without a break in-
between.
1 2 3 4 5 6
B D X C

Again, if the fifth hour is the break, we will have the following sequence.
1 2 3 4 5 6
B D A E X C
However, this is not possible since A cannot
follow D immediately. Hence, the break has to be in the third hour and the arrangement is as
follows:
1 2 3 4 5 6
B D X A E C
26.4 3rd hour is the break.
27.3 B is the first book to be proofread.
28.2 A is to be proofread immediately after the breakfast.
29.4 None of these
30.3 C is to be proofread immediately after E.
For questions 31 to 35:
From (i), we see that he holiday has to be between Monday and Friday (both days included).
Biology has to be done on Thursday. Since Physics cannot immediately preceded biology and
also cannot start the series, it cannot be scheduled on either Wednesday or Sunday. Also,
Physics has to done two days before Astronomy. So, Physics cannot be done on Tuesday. Else
Astronomy would clash with Biology. Therefore, Physics has to be scheduled for Sunday and
thus, Astronomy on Wednesday. Algebra has to be immediately before Geometry. The only
space available for this combination is Friday- Saturday. That leaves Chemistry. Since Sunday
cannot be free, the holiday has to be on Tuesday and Chemistry on Sunday. Therefore, we get
the following as the final table.
Sunday Mond Tuesd Wednes Thursd Frida Saturd
a da a a
y y y y
Chemis Physi X Astrono Biolog Alge Geome
tr my y tr
y y
31.2 Chemistry will start the series of lectures.
32.2 Tuesday is the holiday.
33.1 Physics lectures is one Monday.
34.4 Geometry lecture is on Saturday.
35.2 Biology is scheduled 3 days after Physics.
For questions 36 to 40:
1 – C - S. Sciences x Chemistry … from (iii) 2 – E – Female, Zoology…. From (iv)
3 – G – Female, Physics …. From (iv)
4 – A – Maths not married to Physics teacher …. From (v)
5 – B - Does teach Chemistry or Commerce…. From (vi)
6 – F – Male … from (vii)
8 – 3 females and 4 males, 2 married couples … from (ii)
‘F’ is unmarried; therefore, ‘C is married to ‘D’ …. (using 8) And given that ‘C is married to
Chemistry teacher, therefore.
D is male (using vii) and teaches Chemistry C is female …. [ using (iii) and (vii)]
B is male …. (using 8)
F teaches Commerce …. [using (vi)] Therefore, the final table would look
like as shown below.
Teachers Sex Subjects Married To
A Male Maths E -Zoology
B Male ---- Unmarried
C Femal S. Sciences D -Chemistry
e
D Male Chemistry C --- S.Science
E Femal Zoology A --- Maths
e
F Male Commerce Unmarried
G Femal Physics Unmarried
e
The correct choices are:
36.3
37.4
38.1
40.3
For questions 41 to 45:
Fill up all the absolutes data given. You will get the following table:
Caps Snacks
Amit Blue
Bharti Yellow
Cheryi Sandwich
Deepak
Eric Pizza
Now from (i), red cap and pastries have to be combination, can not fit in anywhere but for
Deepak it fits, since parts e other combinations have filled. That leaves us with two 5 of caps –
green and white and two snacks – ice- and burgers. For caps, Eric does not wear green cap;
tee out of the colours left, he has to wear the white cap. In, Amit does not eat ice-cream;
therefore, he has to eat burger, So, we get the following table.

Caps Snacks
Amit Blue
Bharti Yellow Ice Cream
Cheryi Green Sandwich 41.2 Amit is eating burgers.
Deepa Red Pastries 42.3 Cheryl is wearing the green cap.
k 43.2 Bharati is eating the ice-ream.
Eric White Pizza 44.4 Eric is wearing the white cap.
45.4 ‘Bharati + Burger’ is not the right
combination.

Logical Reasoning
Types of Questions:

 Main Idea
 Inference
 Assumption
 Support /Weaken
 Conclusion
 Logical Techniques
 Logical Error

Types of Question
Now that you’re familiar with the basic principles of Logical Reasoning, let’s look at he most common
types of questions. Certain question types crop up again on the GAT, and it pays to understand them
beforehand.

Main Idea
The test may ask you to identify the main idea of a passage, and it may do this in one of a number of
ways. The main idea can be expressed as
The primary point of the passage ”or” the author’s primary point.”
Most of the time, the main idea will not be directly started in the passage ; you will have to derive it. Be
careful to derive only what is most directly indicated by the passage . A jump of logic will take you
beyond the scope of the passage.
Some other way (but not all he ways) that the test may ask for the main idea questions are which of the
following best expresses the point the author is attempting to make?
The author’s argument is best expressed as………
Which of the following statements best expresses the author’s central point in the preceding passages?
In the preceding passage , the author argues that………..
Example:
Whatever else might be said about Pakistani elections, they are quite unlike those in other countries in
that Pakistanis make choices. And one choice they can make in this free country is to stay home.
What is the author’s point in the preceding passage?
A. Pakistani who do decide to vote make more choices than those who do not.
B. Pakistani elections embody many negative aspects, most of which are not embodied by
elections in other countries.
C. Choosing not to vote is a prerogative of a free citizen.
D. All citizen vote in every elections European countries.
E. Most Pakistani voters are not well informed to vote wisely.

Explanation:
The best response is C.
When you consider the choices, immediately eliminate answers that are irrelevant to the question / or the
major issue of the passage, and answers not at all addressed by the passage. You may eliminate all
choices that do not address the free choice not to vote:
A is irrelevant because it addresses the number of choices rather than the freedom of choice;
B raises issues scarcely addressed in the passage – that is, the negative aspects of elections.
D doesn’t address the issue of choosing not to vote; though it notes that all citizens in other countries
must vote, it neglects the main point that Pakistanis don’t have to;
E is irrelevant to the issue of free choice, stressing instead voter information.
The best choice is C, which addressed the major issue, free choice, and the author’s specific point: the
free choice not to vote.

Inference
This type of logical Reasoning questions asks you to determine an inference or implication in passage.
This type may be expressed more as:
Which of the following can be inferred from the passage?
The author of the passage implies that……
Which of the following inferences can be most reliably drawn from the passage?
What can be validly inferred form the facts and premises expressed in the passage?

Example:
2. We doubt that the latest government report will scare Pakistanis away from beef, chicken, and
Chinese food or that it will empty out the restaurants or cause a run on natural food supplies. If a
diet were to be mandated form Islamabad, Pakistanis probably would order the exact opposite
course. Therefore, the diet that does make sense is to eat a balanced and varied diet composed of
foods form all food groups and containing a reasonable caloric intake.
Which of the following is (are) specifically implied by the passage?
I. Vitamins are necessary to combat disease.
II. A recent report warned of the risks of meat products and imported beverages.
III. Unorthodox suggestions for a more nutritional diet were recently made by the government.
A. I only
B. II only
C. III only
D. I and II only
E. II and III only

Explanation:
The best response is B.
Because the author doubts that Pakistanis will stop eating meats or visiting Bar B Queue, one must
conclude that the author is referring to he latest government report warning of the risks of meat and
Chinese food, statement II. Statement I concerning vitamins may be true but is not specifically implied
other that in a very general sense (nutrition). Statement III is not true: Nothing suggests that the
government report made unorthodox suggestions.

Assumption
In this type of questions, you must determine what assumption lies behind the author’s argument. Other
ways this type may be expressed are:
Which of the following underlies the preceding passage?
The author assumes that…
The preceding argument logically depends on which of the following assumptions?
What is the presupposition of the preceding passage?
Necessary to the preceding reasoning is the assumption that…

Example:
3. After the declaration of company election results, the elected chairperson gave his message to the
workers that, “The responsibility of the management is to serve and not to dominate the workers.”
Which of the following is one basic assumption- underlying chairperson’s statement?
A. The management is capable of dominating the workers.
B. The management chooses to serve rather than dominate the workers.
C. Domination over the workers by the management is a virtue.
D. One must be decisive when facing company union for the first time.
E. The management, preceding the company administration, had been irresponsible.

Explanation:
The best response is A.
Chairperson’s statement is not warranted unless one assumes the capability of the management to
dominate the workers.
A; that assumed capability makes the choice between serving and dominating possible and is thus a basic
assumption.

Support / Weaken:
Which one of the following, if true would most strengthen the argument?
Which one of the following, if true would most seriously damage the argument above?
Which one of the following, if true, casts the most doubt on the argument above?
which one of the following, if true, is the most serious criticism of the argument above?
Strengthen:
Which one of the following, if true would most strengthen the argument?
Which one of the following if true would provide the most support for the conclusion in the argument
above?
The argument above would be more persuasive if which one of the following was found to be true?

Example:
4. Research comparing children of cigarette-smoking parents in Karachi with children of
nonsmoking parents in Hyderabad found that children of smoking parents in Karachi have lower
test scores than do children of nonsmokers in Hyderabad. Therefore, secondhand cigarette smoke
is a cause of the lower test scores.
Which of the following, if true, would weaken the preceding conclusion?
A. Children in Karachi have lower test scores than children in Hyderabad, regardless of whether
their parents smoke or not.
B. More people smoke in Karachi than in Hyderabad.
C. Some children of nonsmoking parents in Sakkhar have good test scores.
D. Nonsmoking parents in Karachi have more children, on average, than those in most other cities.
E. Research has shown that smoking is not only unhealthy for the smoker, but for others in the
nearby vicinity.

Explanation:
The best response is A.
Regardless of weather their parents smoke or not “Children in Karachi have lower test scores than that of
the children in Hyderabad.” Notice that if children in Karachi have lower test scores than children in
Hyderabad, regardless of whether their parents smoke or not, then the cigarette-smoking parents cannot
logically be claimed to be a cause of the lower test scores. This choice would weaken the conclusion.

Reverse Test
To test weather a stamen is necessarily assumed by an author, we can employ the Reverse Test. Simply
deny or negate the statement and see if the argument falls apart. If id does, that choice is a necessary
assumption. If, on the other hand, the argument is unaffected, the choice is wrong.

Example:
Akbar plays cricket for Govt. College Multan. Therefore, Akbar must be over 6 feet tall.
You should recognize the second sentence as the conclusion and the first sentence as the evidence for it.
Do you think? Is the argument complete? Obviously, it is not. The piece that’s missing – the unstated link
between the evidence and conclusion is the assumption, and you could probably pre-phrase this one pretty
easily:
All cricket players for Govt. College Multan are over 6 feet tall.
Is this an assumption really necessary to the argument? Let’s negate it using the Reverse test. What if it is
not true that all cricket players for Govt. College Multan are taller than 6 feet? Can we still logically
conclude that Akbar must be taller than 6 feet? No. we can’t be sure, he might be, but he also might not
be. By denying he statement, then, the argument fal.ls to pieces; it is no longer valid. That’s our
conclusive proof that the statement above is a necessary assumption of this argument.
Use the Reverse test to check quickly whichever choice seems correct.

Conclusion
This type of questions asks for the conclusion that has not yet been stated in the passage.
The following are other ways the test may present this question type:
If the preceding passage is true, then which of the following must necessarily be true?
Which of the following is the best deduction based on the preceding passage?
If the preceding passage is true, which of the following must logically follow?
From the preceding passage, which of the following can reasonably be deducted?
Based on the preceding passage, the author would conclude.

Example
5. After seven years of suggestion from environment departments, the Punjab legislature has finally,
passed an antipollution ordinance. From a reading of the language, the legislation promises to be
one of the most effective bills in the history of the province.
Which of the following can be deducted from the passage?
A. The pollution problem will be reduced in the province.
B. The pollution problem will be eliminated in the province.
C. Pollution will be reduced in the country.
D. Pollution in not now a problem in the country.
E. To reduce pollution, the legislation must now be enforced.

Explanation:
The best response is E.
You are to select the one choice that may necessarily be concluded. Although A and B are possible, they
don’t necessarily have to occur; the ordinance, after all, may not be effective despite its tough language.
Choices C and D are even more remote. But of he choices, E is the safest conclusion that can be drawn.

Logical Technique
This type of questions asks for the technique of reasoning used in the passage. For example, the passage
may use:
A conclusion without adequately supporting it
Contradiction to its original premise within the passage a generalization to prove a specific point an
analogy (a comparison) to further an argument
Here in this type, you need to identify in structural terms how the author has set up the argument.
The test may express this type in these ways:
The author of the passage used which of the following methods of persuasion?
The author makes his point primarily by…..
The author is using what line of reasoning to make the point?
In the preceding passage, the author does which of the following?

Logical Error
This type asks you to find a logical mistake in the reasoning. Pre-reading the question is effective for this
type; once you know you’re looking for an error, instead of wondering why you’re having trouble with
the reasoning, you realize that you have just discovered the flaw.
The preceding conclusion is unsound because the author…
Which of the following inconsistencies seriously undermines the author’s argument?
The reasoning in the preceding passage is flawed because ………..
Which of the following is an inherent error in logic in the preceding passage?

Parallel Line of Reasoning


Not to identify how the author structure the line of reasoning or to identify an error in reasoning, here the
test asks you to “parallel” whatever line of reasoning is presented. That is, you must select the answer
choice that has either the same method of reasoning or the same type of error as the passage.
The test may express this type in these ways:
Which of the following contains a logical flaw similar to the logical flaw in the preceding passage?
The preceding argument exhibits the same principles of inference as which of the following arguments?
Which of the following is logically most similar to the preceding argument?
Which of the following supports its conclusion in the same way as the preceding passage?

Example:
6. Because all poor are intelligent, and all black are intelligent, then all black are poor.
Which of the following parallels the reasoning in the preceding passage?
A. Because all dogs are animals, and cats are animals, then all cats are dogs.
B. All children are human beings, and some men are children, therefore, all men are human beings.
C. Some men are heavy, and some men are tall. Therefore, all tall men are heavy.
D. Some animals are dogs, and some animals are cats. Therefore, all dogs and cats are animals.
E. All cats are animals, and some animals are dogs. Therefore, some animals could be cats or dogs.

Explanation:
The best response is A.
“All dogs are animal, and cats are animal. Therefore, all cats are dogs.”

Question 31-36: Seven airline flights ----- 101,102,103,104,105,106 and 107 ---- are to be scheduled for
departure, one at a time on the hour, from 9:00 a.m until 3:00 p.m. The schedule must conform to the
following requirements:

 Flight 101 must depart at 9:00 a.m.


 Flight 105 must depart later than fight 103, and also later than Flight 102.
 Flights 104. 106 and 107 must depart on consecutive hours in that order.
31. If flight 107 is scheduled to depart at noon, Flight 105 must be scheduled to depart at:

f) 10:00 a.m h) 1:00 p.m j) 3:00 p.m


g) 11:00 a.m i) 2:00 p.m

32. If flights 103 and 104 are scheduled to depart at 11:00 a.m. and 12 noon, respectively, Flight 102
must be scheduled to depart at:

f) 9:00 a.m. g) 10:00 a.m h) 1:00 a.m


i) 2:00 a.m j) 3:00 a.m

33. Which of the following lists three flights in a sequence, from first to last, in which they could be
scheduled to depart consecutively?

f) 101,104,103 h) 104,105,106 j) 106,107,104


g) 102,103,106 i) 106,107,103

34. If Flight 106 is scheduled to depart at 2:00 p.m., Flight 105 must be scheduled to depart at:

f) 10:00 a.m h) 12 noon j) 2:00 p.m


g) 11:00 a.m i) 1:00 p.m

35. Which of the following must be true about the scheduled order of the flights?
f) Flight 103 is scheduled to depart later than Flight 102
g) Flight 104 is scheduled to depart later than Flight 103
h) Flight 105 is scheduled to depart later than Flight 104
i) Flight 106 is scheduled to depart later than Flight 105
j) Flight 107 is scheduled to depart later than Flight 106
36. What is the latest hour at which Flight 102 can be scheduled to depart?

f) 10:00 a.m h) 12 noon j) 2:00 p.m


g) 11:00 a.m i) 1:00 p.m

Question 67-70: Exactly four young wood-carvers at to attend a wood-carving workshop. The four are to
be chosen from a group of seven eligible wood-carvers: three boys --- Frank, George and Harlod ---- and four
girls ---- Juanita, Karla, Mona, and Nellie. The four are to be chosen according to the following conditions:

 If either George or Harold attends the workshop, the other must also attend.
 Juanita and Karla cannot both attend the workshop.
 George and Mona cannot both attend the workshop.
67. Which of the following could be the group attending the workshop?

f) Frank, George, Harold, Mona i) George, Juanita, Mona, Nellie


g) Frank, George, Karla, Nellie j) Juanita, Karla, Mona, Nellie
h) Frank, Karla, Mona, Nellie

68. If Frank and Juanita attend the workshop, which of the following pairs of wood-carvers could be
the others attending?

f) George and Karla h) Harold and Mona


g) George and i) Karla and Mona
Nellie j) Mona and Nellie

69. If Harold attends the workshop, which of the following CANNOT attend?

f) Frank h) Juanita j) Nellie


g) George i) Mona

70. If two wood –carvers of each sex attend the workshops, which of the following is one of the wood-
carvers who must attend?

f) Frank g) Harold h) Juanita


i) Karla
j) Mona
30 IMPORTANT QUESTIONS OF MATHEMATICS
1. 3x + 9 = 18 what is the value of x + 3 ?
A. 3 B. 6
C. −3 D. 36
2. If 5x + 12 = 44 what is the value of 5x − 12 ?
A. 24 B. 32
C. 20 D. 22
3. If 3x + 17 = 9 − x what is the value of x ?
A. 2 B. 3
C.−2 D. −3
4.If x − 5 = 9 what is the value of x2 – 3 ?
A. 193 B. 191
C. 16 D. 11
5. If at – b = c − dt, what is the value of t ?
b−c a
A. B.
a−d b
c b+ c
C. D.
d a+d
1 1 1
6. If x+ x + x=21what is the value of x?
2 4 8
A. 88 B. 44
C. 1 D. 24
7. 2x − 3 = 15 what is the value of (2x – 3)2 ?
A. 81 B. 227
C. ± 225 D. 225
8. 8110 = 3x−7 what is the value of x ?
A. 47 B. 27
C. 51 D. 14
1
9. If = 7 then y = ?
x− y
1 1
A. x+ B. x−¿
7 7
1 x
C. −x D. −1
7 7
10. x = 2t + 5 and y = 4t2 what is the value of y in term of x ?
x−5
A. ¿ )2 B.
2
x−5
C. ( x−5)2 D.
4
11. If x is positive number and x2 + 36 = 100 what is the value of x ?
A. 6 B. 8
C. 114 D. 64
12. If 4 x + 5 = 8 x−1 what is the value of x ?
A. 13 B. 2
C. 14 D. 11

13.If x = 9 then x2 − x = ?√
A. 6561 B. 2530
C. 6552 √
D. 6 − 9
a+3 12
14. = w hat is the positive value of x ?
6 a+ 4
A. 5 B. 12
C. 15 D.18
15. If x and y are positive integers and x2 + 2y2 = 41 , 2x2 + y2 = 34 then x2 = ?
A. 6 B. 8
C. 75 D. 9
16. any positive integer p, $p = p2/3 and £p = 9/p then what is the product of $p and £p ?
A. 2/p B. 1/p
C. p D. 3p
17. If a, b and c are different positive odd integers and a + b + c = 11 what is the greatest positive value of c ?
A. 9 B. 7
C. 3 D. 1
18. If n + 5 = n x 5 , then n = ?
A. 1.25 B. 1.5
C. 0.5 D. 5
a
19. If = 0.75 , then 4a − 3b = ?
b
A. 1 B. 2
C. 0 D. 3
20. Let ab = c, where a , b and c are none zero number, if a is multiplied by 3 and c is divided by 3 this is equivalent b
by:
A. 1/3 B. 3
C. 1/9 D. 9
21. If 5a = 3 , then ( 5a + 3 )2 = ?
A. 9 B. 36
C. 4 D. 25
1
22. If a = , then a3 = ?
3
1 1
A. B.
9 3
3 1
C. D
27 27
23. If 3a – 5 = 7, then what is the value of a?
2
A. B. 4
3
C. 12 D. 2
5w
24. If 4 + = 19 , what is the value of w ?
2
A. 6 B. 30
C. 15 D. 75
25. What is the value of ( 11 − y ) , when 121 − 11y = 77 ?
A. 11 B. 44
C. 7 D. 4
26 What is the value of x2 – 4 , when x6 − 4x4 = 64 and x4 = 4 ?
A. 16 B. 20
C. 12 D. 8
27. If one factor of 8x3 – 27y3 is ( 2x – 3y ) then the other factor is ?
A. 2x + 3y B. 4x2 + 9y2
C. 4x2 + 6xy + 9y2 D. 4x2 + 12xy + 9y2
28. If 32x+y = 16x+2y , then x = ?
A. y B. 5y
y
C. D. 3y
3
29. If px – q = r − sx then , x ?
p+ s r+q
A. B.
r+q p+ s
r−q r −q
C. D.
p+ s p−s
30.If one factor of (a2 – b2 + am + bm ) is ( a + b ) then, the other factor is
A. ( a + b − m ) B. ( a−m )
C. (b−m) D. ( a – b + m )

I.Q
I.Q. TESTS
30. If 12 years are added to 2/3 age of Rani, she will be three years older than today. What is Rani’s
present age?
(a) 25 (b) 27 (c) 26 (d) 29
31. I Drove South for 6 km, then turned left and drove for 2 km, then turned right and drove for 4 km
again turned right and drove 2 km. How far am I from my starting point?
(a) 2 km (b) 4 km (c) 5 km (d) 9 km
32. A man walked 3 km towards North, turned West and walked 2 km, then turned North again and
walked 1 km and then turned East and walked 5 km. How far is he from his starting point?
(a) 4 km (b) 7 km (c) 5 km (d) 9 km
33. A man faces north and covers 7 km, turns west and covers 2 km then turns south and covers 4 km
and turns west again and covers 2km. how far is he from his starting point?
(a) 6 km (b) 7 km (c) 5 km (d) 9 km
34. A and B start from a point x. A goes north and covers 3 km then turns right and covers 4 km. B
goes west and covers 5 km, then turns right and covers 3 km. How far is B from A?
(a) 8 km (b) 11 km (c) 9 km (d) 10 km
35. Point X is in North of point Y and point Y is East of point Z. To which direction is point X with
respect to Z?
(a) North East (b) North (c) East (d) None of the above
36. Point A is located 8 km South of B and C is located 6 km west of A. What is the distance between C
and B?
(a) 8 km (b) 12 km (c) 10 km (d) 9 km
37. You are standing with your face towards east. Then you turn right then again right and then left.
Which direction are you facing at the end?
(a) West (b) South (c) North (d) East
38. I am sixth in a queue numbering from either end. How many are there in the queue?
(a) 11 (b) 9 (c) 7 (d) 13
39. In a cage there are rabbits and pigeons. They have 20 heads and 48 feet. How many rabbits are
there?
(a) 6 Rabbits (b) 4 Rabbits (c) 8 Rabbits (d) 10 Rabbits
40. A clock seen through a mirror reads quarter past three. What is the correct time?
(a) Quarter to nine (b) 15 past 9 (c) 15 to 3 (d) None of the above
41. If a man stands upside down with his face towards south, where will be his left hand point?
(a) East (b) West (c) North (d) South
42. A is the father of B but B is not the son of A. What is B to A?
(a) Daughter (b) Father (c) Mother (d) None of these
43. A party consisted of a man and his wife, their two sons and their wives and four children of each
son. How many persons were there in all the party?
(a) 24 (b) 14 (c) 34 (d) 16
44. A boy had twice as many sums wrong as right. If he had 24 sums in all how many of them were
wrong?
(a) 18 (b) 20 (c) 16 (d) 15
45. The old man’s son is my son’s uncle. What is the old man to me?
(a) Brother (b) Father (c) Uncle (d) None of above
46. A is the cousin of the father of B. What is the relationship between B and the son of A?
(a) Nephew (b) Cousin (c) Niece (d) None of above
47. If Sunday dawned three days before yesterday what day will dawn two days after tomorrow?
(a) Sunday (b) Monday (c) Tuesday (d) Wednesday
48. Reaching a party day before yesterday I found myself two days late. If day after tomorrow is
Friday, what day was the party scheduled to be held?
(a) Sunday (b) Monday (c) Saturday (d) Thursday
49. If X and Y are parents of Z. But Z is not the son of X. What is Z to X?
(a) Nephew (b) Daughter (c) Father (d) Aunt
50. Your father says “I have four sons” but you say “I have only three brothers”. Who is wrong?
(a) Father (b) Son (c) Both (d) None
51. Find the missing number:
B 5 L ?
2 E 12 W
(a) 25 (b) 23 (c) 21 (d) 19
52. Find the missing number: 5, 6 , 9, 20, 10, 7, 8, 9, 24 ?
(a) 22 (b) 12 (c) 18 (d) 16
53. What is the missing number? 2,4,2,3,9,6,4,16,?
(a) 10 (b) 12 (c) 14 (d) 8
54. Find the missing number: 10, 18, 26, 40, ?
(a) 50 (b) 52 (c) 54 (d) 56
55. Find the missing number? 5,7,12,24,6,8,14,28,9,5,14,?
(a) 26 (b) 24 (c) 22 (d) 28
56. Find the missing number 3,4,49,5,4,?
(a) 81 (b) 71 (c) 61 (d) 91
57. Find the missing number? 5, 9, 16, 29, 54,?

(a) 105 (b) 107 (c) 103 (d) 104


29. 9, 18, 19, 38, 75, ?
(a) 64 (b) 76 (c) 140 (d) 150
30. A is the brother of B but B is not the brother of A. What is B to A?
(a) Nephew (b) Sister (c) Mother (d) None of above
31. In a class of 33, the boys outnumber the girls by 7. How many girls are there?
(a) 13 (b) 26 (c) 40 (d) 47
32. If X is west of Y and Y is north of Z; towards which direction of X is Z?
(a) East (b) South East (c) South West (d) North East
33. There are four numbers, Average of the first three is 15 and that of the last three is 16. If the last
number is 19, find the first number?
(a) 16 (b) 18 (c) 20 (d) 21
34. If you start from pt A and walk 5 km towards the west, then turn right and walk 4 km towards the
north, then turn right again and walk 3 km. You are going in which direction?
(a) North (b) South (c) West (d) East
35. A man faces south and walks 10 km, turns west and covers 4 km, then turns north and cover 7 km.
How far is he from the starting point?
(a) 4 km (b) 5 km (c) 6 km (d) 8 km
36. If the doctor gave you 4 tablets and advised you a take tablet every half an hour, how long will you
take to eat all tablets?
(a) 2 hours (b) 1 ½ hours (c) 2 ½ hours (d) None of above
37. It takes 2 minutes to boil a single egg. How many minutes will it take to boil 5 eggs together?
(a) 2 minutes (b) 5 minutes (c) 8 minutes (d) 10 minutes
38. If Ali stood on the top step of a ladder and there were six more steps the midway step, how many
steps in all did ladder comprise of?
(a) 10 (b) 15 (c) 14 (d) 16
39. How many different three-digit numbers can be formed by 0, 3 and 5, if none of them is repeated
in a number:
(a) 2 (b) 3 (c) 4 (d) 5

LECTURE 5
ANALYTICAL REASONING

CSS G. ABILITY & MATHS PAPER 2018 SECTION II Q

(Abi type krna hai)


GRE Maths Done (Sir Sheraz)

Analytical Reasoning Done (Sir Sheraz)

Analytical Reasoning
Question 1-3:From exactly seven objects – R, S, T, U, X, Y, and Z – a group of exactly four objects
must be selected in accordance with the following conditions:
● If R is selected, T must also be selected
● If S is selected, U must also be selected
● If X and Y are both selected together, T cannot be selected.
58. If X and Y are both selected together, which of the following must also be selected?
(a) R (b) S (c) T (d) U
(e) Z
59. If S and Z are both selected together, each of the following could also be selected EXCEPT.
(a) R (b) T (c) U (d) X
(e) Y
60. If U is not selected, which of the following can be, but does not have to be selected?
(a) R (b) S (c) T (d) X
(e) Z
Question 4-9:A certain group of eight people is made up of two women – S and T; three men – U,V, and
W; and three children – X, Y and Z. Each of the eight must be on one, and only one, of three teams being
formed according to the following conditions.
● Each team must have either two or three members.
● Each team must have at least one child.
● S and X cannot be on the same team.
● V and W cannot be on the same team.
● U and Y must be on the same team.
61. If U on the same team as V, which of the following pairs must be on a team together.
(a) S and Y (b) S and Z (c) T and W (d) T and Y
(e) W and X
62. If is on the same team as Y, which of the following could be true?
(a) U is on a team with only on other member.
(b) T is on a team with only one other member.
(c) T is on the same team as V. (d) W is on the same team as Y.
(e) Z is on the same team as U.
63. If the two women are on a team together, which of the following must be on a team, which only one
other member?
(a) U (b) V (c) W (d) X
(e) Z
64. If two of the men are on a team together, which of the following must be on a team that has three
members?
(a) S (b) T (c) V (d) W
(e) Y
65. If S is on the same team as U, which of the following could be true?
(a) The two women are on a team together (b) Two of the men are on a team together.
(c) S is on a team with V (d) T Is on a team with Z
(e) U is on a team with W.
66. If one man is on each team, any of the following pairs could be together on a team EXCEPT:
(a) S and T (b) S and W (c) T and U (d) T and W
(e) V and Z
Directions:Each question or group of questions is based on a passage or set of conditions.
In answering some of the questions. It may be useful to draw a rough diagram.
For each questions, selected the best answer choice given.
Question 10-13: Six products – U, V, W, X, Y and Z – are to be placed in the display window of a
vending machine with six compartments. Numbered 1 through 6 form left to right. The products must be
placed in the window, one product in each compartment, according to the following conditions.
● U cannot be immediately to the left or immediately to the right of V.
● W must be immediately to the left of X.
● Z cannot be in compartment 6.
67. Which of the following products CANNOT be placed in compartment 1?
(a) U (b) V (c) W (d) X
(e) Z
68. If X is placed in compartment 3, W must be placed in compartment .
(a) 1 (b) 2 (c) 4 (d) 5
(e) 6
69. If U is placed in compartment 5, which of the following products must be placed in compartment
6?
(a) V (b) W (c) X (d) Y
(e) Z
70. If Z is placed in compartment 3, immediately to the right of X, which of the following products
must be placed in compartment 5?
(a) U (b) V (c) W (d) X
(e) Y
Question 14-20:The manager of a commercial printing firm is scheduling exactly six jobs - P, Q, S, T,
W, and X – for a particular week. Monday through Saturday. Each job cab be completed in one full day
and exactly one job will be completed in one full day and exactly one job will be scheduled for each
day . the jobs must be scheduled according to the following conditions.
● P must be printed sometime before S is printed.
● T must be printed don the day immediately before or the day immediately after the day on which X
is printed.
● W must be printed on Thursday.
71. Which of the following is and acceptable schedule of jobs for the week?
Mon Tues Wed Thurs Fri Sat

(A) P Q T W X S

(B) P W S X T Q

(C) Q X T W S P

(D) T X P W Q S

(E) X P T W S Q

72. Any of the following could be printed on Saturday EXCEPT:


(a) P (b) Q (c) S (d) T
(e) X
73. If Q is printed on Wednesday, which of the following could be true?
(a) P is printed on Tuesday (b) S is printed on Monday
(c) S is printed on Friday (d) T is printed on Monday
(e) X is printed on Thursday
74. If X is printed on Monday, which of the following must be true?
(a) P is printed sometime before Q (b) P is printed sometime before W
(c) Q is printed sometime before S (d) W is printed sometime before Q
(e) W is printed sometime before S
75. If P is printed on Tuesday, which of the following must be true?
(a) Q is printed on Monday (b) S is printed on Thursday
(c) S is printed on Saturday (d) T is printed on Friday
(e) X is printed on Saturday
76. If T is printed on Tuesday, any of the following could be true EXCEPT:
(a) P is printed on Monday (b) Q is printed on Saturday
(c) S is printed on Wednesday (d) S is printed on Friday
(e) X is printed on Wednesday
77. If Q is printed on Friday, Which of the following must be true?
(a) P is printed on Monday (b) P is printed on Wednesday
(c) S is printed on Saturday (d) T is printed on Monday
(e) X is printed on Tuesday
Question 21-25:A florist has exactly seven varieties of flowers ----- P, Q, R, S, T, U, and V ---- from
which she must select combinations of exactly five varieties with which to make flower arrangements.
Any combination of the five varieties that conforms to all of the following conditions is acceptable.
● If P is used in an arrangement, T cannot be used in that arrangement.
● If Q is used in an arrangement, U must also be used in that arrangement.
● If R used in an arrangement, T must also be used in that arrangement.
78. Which of the following is an acceptable combination of varieties that the florist can select for and
arrangement?
(a) P,Q, S, T, U (b) P, Q, R, U ,V (c) P, S, T, U, V (d) Q, R, S, U, V
(e) Q, R, S, T, U
79. If the florist selects variety R to be included in an arrangement, which of the following must be
true of that arrangement?
(a) P is not used (b) U is not used (c) Q is used (d) S is used
(e) V is used.
80. If variety P is used in an arrangement, which of the following CANNOT be sued in that
arrangement?
(a) Q (b) R (c) S (d) U
(e) V
81. If the florist does not select variety V for an arrangement, which of the following also CANNOT be
selected?
(a) P (b) Q (c) R (d) S
(e) T
82. Which of the following substitutions cab the florist always make without violating the conditions
governing flower combination, provided the variety mentioned first was not, and the variety
mentioned second was, originally going to be used in the arrangement concerned.
(a) P for R (b) Q for U (c) R for T (d) S for V
(e) V for T
Questions 26-31: Five ships --- J, K, L, M and N --- are to unloaded on 5 consecutive days beginning on
Monday and ending on Friday according to the following conditions:
● Each ship takes exactly one day to unload.
● K must be unloaded on a day preceding the days M and N are unloaded.
● L cannot be unloaded on Tuesday.
● M Must be the second ship unloaded after J is unloaded.
83. If M is unloaded on Friday, which of the following must be true?
(a) J is unloaded on Wednesday (b) K is unloaded on Tuesday
(c) L is unloaded on Monday (d) L is unloaded on Thursday
(e) N is unloaded on Thursday
84. If K, M, and N are to be unloaded one immediately after the other in the that order, the two days
on which J can be unloaded are:
(a) Monday and Tuesday (b) Monday and Friday
(c) Tuesday and Wednesday (d) Wednesday and Friday
(e) Thursday and Friday
85. If L in unloaded on the day immediately after the day J is unloaded, which of the following must
be true?
(a) J is unloaded on Wednesday (b) K is unloaded on Monday
(c) L is unloaded on Thursday (d) M is unloaded on Friday
(e) N is unloaded on Tuesday
86. If J is unloaded on Monday, which of the following must be true?
(a) L is unloaded on Tuesday (b) L is unloaded before M
(c) K is unloaded on Tuesday (d) L is unloaded on Thursday
(e) N is unloaded on Thursday

87. N cab be unloaded any day of the week EXCEPT:


(a) Monday (b) Tuesday (c) Wednesday (d) Thursday
(e) Friday
88. On which of the following days can any one of the five ships be unloaded?
(a) Monday (b) Tuesday (c) Wednesday (d) Thursday
(e) Friday
Questions 32-35:A contractor will build five houses in certain town on a street that currently has no
house on it. The contractor will select from seven different models of house ---- T, U, V, W, X, Y, and Z
the town’s planning board has placed the following restrictions on the contractor.
● No model can be selected for more that one house.
● Either model W must be selected to model Z must be selected, but both cannot be selected.
● If model Y is selected, then model V must also be selected
● If model U is selected, then model W cannot be selected.
89. If model U is one of the models selected for the street, the n which of the following models must
also be selected?
(a) T (b) W (c) X (d) Y
(e) Z
90. If T, U, and X are three of the models selected for the selected for the street, then which of the
following must be the other two models selected?
(a) V and W (b) V and Y (c) V and Z (d) W and Y
(e) Y and Z
91. Which of the following is an acceptable combination of models that can be selected for the street?
(a) T, U, V, X, Y (b) T, R, X, Y, Z (c) T, V, X, Y, Z (d) U, V, W, X, Y
(e) V, W, X, Y, Z
92. If model Z is one model not selected for the street, then the other model NOT selected must be
which of the following?
(a) T (b) U (c) V (d) W
(e) X
Questions 36-39: A group of three objects must be selected form six objects --- K, O, S, T, V and W ---
according to the following.
● K or S or both must be selected
● V or O must be selected
● Neither V nor S will selected with O
93. Which of the following is an acceptable selection of objects?
(a) K, O, and S (b) K, S and T (c) K, S, and V (d) O, S and V
(e) O, T, and V
94. Which of the following pairs of objects CANNOT both the amount the objects selected?
(a) K and O (b) K and T (c) O and W (d) T and W
(e) V and W

95. If S is selected, which of the following must also be among the objects selected?
(a) K (b) O (c) T (d) V
(e) W
96. If V is not selected, which pair of objects must be among those selected?
(a) K and O (b) K and T (c) K and W (d) O and T
(e) O and W
Questions 40-42: Exactly seven people --- Q, R, S, T, X, Y and Z ---- serve on an advisory board.
Q,R,S, and Z and have been elected to the board, and X, Y, and Z have been appointed to the board.
Three person or four person panels are sometimes drawn from he board to study proposals. Each panel
must include at last one elected and at least one appointed board member, but no panel can consist of
equal numbers of elected and appointed members. Each panel is chaired by a person who is member of
the group of board members (elected or appointed) whose representatives are in the minority on that
panel. Any panel must also conform to the following conditions:
● If Q serves on a panel, T cannot serve on that panel.
● If R serves on a panel, X cannot serve on that panel.
● T and Y cannot serve on a panel unless they serve together.
● If Z serves on a panel, X must also serve on the panel.
97. Which of the following could be a panel drawn from the advisory board?
(a) Q,R,S (b) S,X,Z (c) T,Y,Z (d) Q,S,T,Y
(e) R,T,X,Y
98. If R serves on a panel, it must be true that:
(a) It is a three person panel (b) It is a four person panel
(c) R chairs the panel (d) T Chairs the panel
(e) Y chairs the panel
99. Each of the following could chair a panel EXCEPT:
(a) S (b) T (c) X (d) Y
(e) Y
Question 43-48: A college has decided to appoint a four member committee to review educational
policy. Eight faculty members are eligible to serve on the committee: F, G, J , K, L, N, O and P. three of
these eight --- G, L, and N ---- are untenured, whereas the remaining five are tenured, college policy and
the personalities of the eight people in question dictate that the choice of committer member meet the
following conditions:
● The committee must be composed of exactly two untenured faculty members.
● G and J cannot serve together.
● If P is on the committee, then neither N nor O can be on the committee.
● If K is on the committee, then J must be on the committee,
● If both F and J are on the committee, then L cannot be on the committee.
100. If P is on the committee, the other three members of the committee must be:
(a) F,G and L (b) F,G and N (c) G,J and L (d) G, K, and L
(e) J, K, and L
101. If J is on the committee, which of the following is one of the faculty members who must also be on
the committee?
(a) F (b) K (c) L (d) O
(e) P
102. If G is on the committee, any of the following could be on the committee EXCEPT:
(a) K (b) L (c) N (d) O
(e) P
103. If neither F nor P is on the committee, which of the following could be list of the committee
members?
(a) G, J, L, and O (b) G, K, N, and O (c) G, L, O, and K (d) J, K, L, and O
(e) J, L, N and O
104. If L is not on the committee, which of the following is a list of people all of whom must be on the
committee?
(a) F,G, and N (b) F,J, and N (c) G, K, and N (d) J, K, and N
(e) J, N and P
105. If L and N do not both server on the committee, which of the following is a list of people all of the
whom could be on the committee together?
(a) F,G, and L (b) F,J, and N (c) F,N, and P (d) G,K and N
(e) K,L and P
Question 49-55: Six puppies --- K, L, M, S, T and U --- must each be scheduled for examination by a
veterinarian. The puppies are to be examined on at time in six consecutive time slots on the same day
according to the following conditions:
● M Cannot be examined immediately before or immediately after S.
● L must be examined immediately before U.
● K must be examined fourth.
106. Which of the following is an acceptable examination schedule for the puppies, in order from first
examined last examined:
(a) L, T, S, K, M, U (b) L, U, T, K, S , M (c) M, T, S, L, U, K (d) S, T, M, K, L, U
(e) T, M, S, K, L, U
107. If L is examined second which of the following must be true?
(a) K is examined at some time before S (b) M is examined at some time before T
(c) T is examined at some time after K (d) M is examined sixth
(e) S is examined first
108. S can be examined in any of the following time slots EXCEPT:
(a) First (b) Second (c) Third (d) Fifth
(e) Sixth
109. If L is examined first, T must be examined:
(a) Immediately before K (b) Immediately before S
(c) At some time before M (d) At some time after K
(e) At some time after S
110. If S examined sixth, which of the following is a complete and accurate list of the time slots any one
of which could be the time slot in which M is examined?
(a) First (b) First, second (c) First, third (d) First, second, third
(e) Second, third, fifth
111. If U is examined at some time before M is examined , L can be examined.
(a) Immediately after S (b) Immediately after T
(c) Immediately before T (d) At some time after M
(e) At some time after K
112. If both M and T are examined at some time after K is examined, S must be examined:
(a) First (b) Second (c) Third (d) First or else third
(e) Second or else third
Question 56-62: At an art show, exactly five sculptures are to be displayed in a room with exactly five
stands, arranged along a single wall and numbered consecutively I through 5. The sculptures are to be
selected from a total of eight sculptures ---- M, N, P, Q, R, S, T, and U ---- and displayed one sculpture
on each stand, according to the following conditions:
● Either M or U both must be selected.
● I f M is selected, M must be on stand 1.
● Either R or S must be on stand 3.
● If T is selected , P must also be selected, and T and P must then be on stands are immediately
adjacent to one another.
113. Which of the following is an acceptable selection of sculptures to be displayed on stands I through
5?
1 2 3 4 5
(A) M R T P Q
(B) N T S U Q
(C) P T S R U
(D) T P R S M
(E) U N Q P T
114. If S is on stand 1, which of the following must be true?
(a) P is on stand 4 (b) R is on stand 3 (c) T is on stand 2 (d) T is on stand 4
(e) U is on stand 5
115. If T is on stand 5, which of the following pairs of sculptures can be on stands that are immediately
adjacent to each other?
(a) M and P (b) Q and N (c) Q and P (d) R and T
(e) U and R
116. If U is on stand 4, any of the following can be on stand 5 EXCEPT:
(a) N (b) P (c) Q (d) R
(e) T
117. If T is on stand 2, which of the following sculptures must be selected?
(a) M (b) N (c) R (d) S
(e) U
118. If P is not selected and R is on stand 1, which of the following lists, in alphabetical order, those
sculpture’s that must also be selected?
(a) M,Q, T, and U (b) N, Q, S, and T (c) N, Q, S, and U (d) N, S, T, and U
(e) Q, S, T, and U
119. If Q is displayed on a stand immediately adjacent to a stand on which R is displayed and
immediately adjacent to a stand on which S is displayed, which of the following must be true?
(a) N is on either stand 4 or stand 5 (b) Q is on either stand 2 or stand 4
(c) R is on either stand 1 or stand 3 (d) S is on either stand 3 or stand 5
(e) U is on either stand 2 or stand 4
Question 63-66: Seven meetings --- J, K, L, M, N, O and P ---- are to be schedules, one on each day of a
week that begins on Sunday.
● Meeting J must take place on Sunday.
● Meeting K must take place after both meeting L and Meeting M.
● Meeting N, O and P must take place on three consecutive days, not necessarily in that order.
120. Which is the latest day of the week on which meeting L can take place?
(a) Tuesday (b) Wednesday (c) Thursday (d) Friday
(e) Saturday
121. Which of the following must be true about the order of meeting?
(a) L takes place after J (b) L takes place after O
(c) N takes place after O (d) P takes place after P
(e) takes place after P
122. If meeting O is on Saturday, then meeting K must take place on:
(a) Monday (b) Tuesday (c) Wednesday (d) Thursday
(e) Friday
123. Which of the following represents a possible ordering of meetings on three consecutive days?
(a) JMK (b) KLO (c) MNJ (d) OJN
(e) POM
Question 67-71:Seven dignitaries --- F, G, H, I, N, O and P ---- are to be seated together at a diplomatic
ceremony. They will be seated in a row of seven chairs, numbered form 1 to 7, form front to back. Any
seating is acceptable as long as all seven dignitaries are seated, one in each chair, and the seating
conforms to the following rules:
● F must sit in the chair immediately O’s chair.
● G cannot sit in the chair immediately in form of N’s Chair, and G cannot sit in the chair
immediately behind N’s Chair.
● There must be exactly two chairs between the chairs of H and P.
● There must be at least one chair between the chairs of I and P.
● N must sit in chair 3.
124. Which of the following seating arrangements, form chair 1 through chair 7, conforms to the rules?
(a) F, I, N, P, G, O, H (b) G, P, N, I, H, O, F
(c) I, G, N, P, O, F, H (d) I, H, N, P, O, F, G
(e) O, F, H, N, I, P, G
125. If F sits in chair 6 and H sits in chair 7, which of the following dignitaries must sit in chair 2?
(a) G (b) I (c) N (d) O
(e) P
126. If the seating arrangement, from chair 1 through chair 7, is G, I, N, H, O, F, P, which of the
following Paris of dignitaries can exchange seats without violating the rules?
(a) F and G (b) G and H (c) G and I (d) H and P
(e) I and P
127. If O sits in chair 1 and H sits in chair 7, then the number of chairs between F’s chair and I’s chair
must be:
(a) Zero (b) One (c) Two (d) Three
(e) Four
128. If H sits chair 4 and F sits in chair 6, then the dignitaries in chairs 1 and 7, respectively, must be:
(a) G and O (b) G and P (c) I and P (d) O and I
(e) P and O

ANSWER
1 d 2 a 3 d 4 b 5 b 6 d 7 e
8 e 9 a 1 d 1 b 1 d 1 e 1 d

1 a 1 d 1 e 1 a 1 c 2 c 2 e

2 a 2 b 2 a 2 d 2 a 2 a 2 b

2 c 3 a 3 c 3 e 3 c 3 c 3 b

3 c 3 d 3 d 3 a 4 b 4 e 4 e

4 a 4 c 4 a 4 e 4 a 4 a 4 d

5 c 5 b 5 d 5 c 5 a 5 d 5 c

5 b 5 e 5 e 6 e 6 c 6 b 6 d

6 a 6 c 6 e 6 b 6 b 6 d 7 d

7 b
Model question: Read the information given carefully and answer the following questions.
Amit, Bharati, Cheryl, Deepak and Eric are five fiends sitting in a restaurant. They are wearing
caps {five different colours – yellow, blue, green, white and red. Also, they are eating five
different snacks burgers, sandwiches, ice-cream, pastries and pizza.
V. The person wearing a red cap is eating pastries.
VI. Amit does not eat ice-cream and Cheryl is eating sandwiches.
VII. Bharati is wearing a yellow cap and Amit is wearing a blue cap.
VIII. Eric is eating pizza and is not wearing a green cap.
1. What is Amit eating?
2. Who is wearing green cap?
3. Who is eating ice-cream?
4. Which color cap Eric wearing?
5. Which of the following is not correct?
a. Yellow+ ice cream
b. Red + pastry
c. White + pizza
d. Bharti + burger

Question 31-36: Seven airline flights ----- 101,102,103,104,105,106 and 107 ---- are to be
scheduled for departure, one at a time on the hour, from 9:00 a.m until 3:00 p.m. The schedule
must conform to the following requirements:
 Flight 101 must depart at 9:00 a.m.
 Flight 105 must depart later than fight 103, and also later than Flight 102.
 Flights 104. 106 and 107 must depart on consecutive hours in that order.
37. If flight 107 is scheduled to depart at noon, Flight 105 must be scheduled to depart at:

k) 10:00 a.m m) 1:00 p.m o) 3:00 p.m


l) 11:00 a.m n) 2:00 p.m

38. If flights 103 and 104 are scheduled to depart at 11:00 a.m. and 12 noon, respectively,
Flight 102 must be scheduled to depart at:

k) 9:00 a.m. m) 1:00 a.m o) 3:00 a.m


l) 10:00 a.m n) 2:00 a.m

39. Which of the following lists three flights in a sequence, from first to last, in which they
could be scheduled to depart consecutively?

k) 101,104,103 m) 104,105,106 o) 106,107,104


l) 102,103,106 n) 106,107,103
40. If Flight 106 is scheduled to depart at 2:00 p.m., Flight 105 must be scheduled to depart
at:

k) 10:00 a.m m) 12 noon o) 2:00 p.m


l) 11:00 a.m n) 1:00 p.m

41. Which of the following must be true about the scheduled order of the flights?
k) Flight 103 is scheduled to depart later than Flight 102
l) Flight 104 is scheduled to depart later than Flight 103
m) Flight 105 is scheduled to depart later than Flight 104
n) Flight 106 is scheduled to depart later than Flight 105
o) Flight 107 is scheduled to depart later than Flight 106
42. What is the latest hour at which Flight 102 can be scheduled to depart?

k) 10:00 a.m m) 12 noon o) 2:00 p.m


l) 11:00 a.m n) 1:00 p.m

Question 67-70: Exactly four young wood-carvers at to attend a wood-carving workshop. The
four are to be chosen from a group of seven eligible wood-carvers: three boys --- Frank, George
and Harlod ---- and four girls ---- Juanita, Karla, Mona, and Nellie. The four are to be chosen
according to the following conditions:
 If either George or Harold attends the workshop, the other must also attend.
 Juanita and Karla cannot both attend the workshop.
 George and Mona cannot both attend the workshop.
71. Which of the following could be the group attending the workshop?

k) Frank, George, Harold, Mona n) George, Juanita, Mona, Nellie


l) Frank, George, Karla, Nellie o) Juanita, Karla, Mona, Nellie
m) Frank, Karla, Mona, Nellie

72. If Frank and Juanita attend the workshop, which of the following pairs of wood-carvers
could be the others attending?

k) George and m) Harold and o) Mona and


Karla Mona Nellie
l) George and n) Karla and Mona
Nellie

73. If Harold attends the workshop, which of the following CANNOT attend?

k) Frank m) Juanita o) Nellie


l) George n) Mona
74. If two wood –carvers of each sex attend the workshops, which of the following is one of
the wood-carvers who must attend?

k) Frank
l) Harold
m) Juanita
n) Karla
o) Mona

Question 10-13: Six products – U, V, W, X, Y and Z – are to be placed in the display window of
a vending machine with six compartments. Numbered 1 through 6 form left to right. The
products must be placed in the window, one product in each compartment, according to the
following conditions.
 U cannot be immediately to the left or immediately to the right of V.
 W must be immediately to the left of X.
 Z cannot be in compartment 6.
10. Which of the following products CANNOT be placed in compartment 1?

F. U H. W J. Z
G. V I. X

11. If X is placed in compartment 3, W must be placed in compartment.

F. 1 H. 4 J. 6
G. 2 I. 5

12. If U is placed in compartment 5, which of the following products must be placed in


compartment 6?

F. V H. X J. Z
G. W I. Y

13. If Z is placed in compartment 3, immediately to the right of X, which of the following


products must be placed in compartment 5?

F. U H. W J. Y
G. V I. X
Question 21-25: A florist has exactly seven varieties of flowers ----- P, Q, R, S, T, U, and V ----
from which she must select combinations of exactly five varieties with which to make flower
arrangements. Any combination of the five varieties that conforms to all of the following
conditions is acceptable.
 If P is used in an arrangement, T cannot be used in that arrangement.
 If Q is used in an arrangement, U must also be used in that arrangement.
 If R used in an arrangement, T must also be used in that arrangement.
21. Which of the following is an acceptable combination of varieties that the florist can select
for and arrangement?
F. P, Q, S, T, U
G. P, Q, R, U, V
H. P, S, T, U, V
I. Q, R, S, U, V
J. Q, R, S, T, U
22. If the florist selects variety R to be included in an arrangement, which of the following
must be true of that arrangement?

F. P is not used I. S is used


G. U is not used J. V is used.
H. Q is used

23. If variety P is used in an arrangement, which of the following CANNOT be sued in that
arrangement?

F. Q H. S J. V
G. R I. U

24. If the florist does not select variety V for an arrangement, which of the following also
CANNOT be selected?

F. P H. R J. T
G. Q I. S

25. Which of the following substitutions cab the florist always make without violating the
conditions governing flower combination, provided the variety mentioned first was not,
and the variety mentioned second was, originally going to be used in the arrangement
concerned.

F. P for R H. R for T J. V for T


G. Q for U I. S for V

Questions 26-31: Five ships --- J, K, L, M and N --- are to unloaded on 5 consecutive days
beginning on Monday and ending on Friday according to the following conditions:
 Each ship takes exactly one day to unload.
 K must be unloaded on a day preceding the days M and N are unloaded.
 L cannot be unloaded on Tuesday.
 M Must be the second ship unloaded after J is unloaded.
26. If M is unloaded on Friday, which of the following must be true?
F. J is unloaded on Wednesday
G. K is unloaded on Tuesday
H. L is unloaded on Monday
I. L is unloaded on Thursday
J. N is unloaded on Thursday
27. If K, M, and N are to be unloaded one immediately after the other in the that order, the
two days on which J can be unloaded are:
F. Monday and Tuesday
G. Monday and Friday
H. Tuesday and Wednesday
I. Wednesday and Friday
J. Thursday and Friday
28. If L in unloaded on the day immediately after the day J is unloaded, which of the
following must be true?

F. J is unloaded on Wednesday I. M is unloaded on Friday


G. K is unloaded on Monday J. N is unloaded on Tuesday
H. L is unloaded on Thursday

29. If J is unloaded on Monday, which of the following must be true?


F. L is unloaded on Tuesday
G. L is unloaded before M
H. K is unloaded on Tuesday
I. L is unloaded on Thursday
J. N is unloaded on Thursday
30. N cab be unloaded any day of the week EXCEPT:

F. Monday H. Wednesday J. Friday


G. Tuesday I. Thursday

31. On which of the following days can any one of the five ships be unloaded?

F. Monday H. Wednesday J. Friday


G. Tuesday I. Thursday
Questions 32-35: A contractor will build five horses in certain town on a street hat currently has
no house on it. The contractor will select from seven different models of house ---- T, U, V, W,
X, Y, and Z the town’s planning board has placed the following restrictions on the contractor.
 No model can be selected for more that one house.
 Either model W must be selected to model Z must be selected, but both cannot be
selected.
 If mode Y is selected, then model V must also be selected
 If model U is selected, then model W cannot be selected.
32. If model U is one of the models selected for the street, the n which of the following
models must also be selected?

F. T H. X J. Z
G. W I. Y

33. If T, U, and X are three of the models selected for the selected for the street, then which
of the following must be the other two models selected?

F. V and W H. V and Z J. Y and Z


G. V and Y I. W and Y

34. Which of the following is an acceptable combination of models that can be selected for
the street?

F. T, U, V, X, Y H. T, V, X, Y, Z J. V, W, X, Y, Z
G. T, R, X, Y, Z I. U, V, W, X, Y

35. If model Z is one model not selected for the street, then the other model NOT selected
must be which of the following?

F. T H. V J. X
G. U I. W

Questions 36-39: A group of three objects must be selected form six objects --- k, O, S, T, V and
W --- according to the following.
36. Which of the following is an acceptable selection of objects?

F. K, O, and S I. O, S and V
G. K, S and T J. O, T, and V
H. K, S, and V

37. Which of the following pairs of objects CANNOT both the amount the objects selected?

F. K and O H. O and W J. V and W


G. K and T I. T and W
38. If S is selected, which of the following must also be among the objects selected?

F. K H. T J. W
G. O I. V

39. If V is not selected, which pair of objects must be among those selected?
F. K and O
G. K and T
H. K and W
I. O and T
J. O and W

Question 49-55: Six puppies --- K, L, M, S, T and U --- must each be scheduled for examination
by a veterinarian. The puppies are to be examined on at time in six consecutive time slots on the
same day according to the following conditions:
 M Cannot be examined immediately before or immediately after S.
 L must be examined immediately before U.
 K must be examined fourth.
49. Which of the following is an acceptable examination schedule for the puppies, in order
from first examined last examined:
F. L, T, S, K, M, U
G. L, U, T, K, S , M
H. M, T, S, L, U, K
I. S, T, M, K, L, U
J. T, M, S, K, L, U
50. If L is examined second which of the following must be true?
F. K is examined at some time before S
G. M is examined at some time before T
H. T is examined at some time after K
I. M is examined sixth
J. S is examined first
51. S can be examined in any of the following time slots EXCEPT:

F. First H. Third J. Sixth


G. Second I. Fifth

52. If L is examined first, T must be examined:


F. Immediately before K
G. Immediately before S
H. At some time before M
I. At some time after K
J. At some time after S
53. If S examined sixth, which of the following is a complete and accurate list of the time
slots any one of which could be the time slot in which M is examined?
F. First
G. First, second
H. First, third
I. First, second, third
J. Second, third, fifth
54. If U is examined at some time before M is examined , L can be examined.
F. Immediately after S
G. Immediately after T
H. Immediately before T
I. At some time after M
J. At some time after K
55. If both M and T are examined at some time after K is examined, S must be examined:
F. First
G. Second
H. Third
I. First or else third
J. Second or else third

Question 56-62: At an art show, exactly five sculptures are to be displayed in a room with
exactly five stands, arranged along a single wall and numbered consecutively I through 5. The
sculptures are to be selected from a total of eight sculptures ---- M, N, P, Q, R, S, T, and U ----
and displayed one sculpture on each stand, according to the following conditions:
 Either M or U both must be selected.
 If M is selected, M must be on stand 1.
 Either R or S must be on stand 3.
 If T is selected, P must also be selected, and T and P must then be on stands are
immediately adjacent to one another.
56. Which of the following is an acceptable selection of sculptures to be displayed on stands
I through 5?

1 2 3 4 5
(A) M R T P Q
(B) N T S U Q
(C) P T S R U
(D) T P R S M
(E) U N Q P T
57. If S is on stand 1, which of the following must be true?
F. P is on stand 4
G. R is on stand 3
H. T is on stand 2
I. T is on stand 4
J. U is on stand 5
58. If T is on stand 5, which of the following pairs of sculptures can be on stands that are
immediately adjacent to each other?
F. M and P H. Q and P J. U and R
G. Q and N I. R and T

59. If U is on stand 4, any of the following can be on stand 5 EXCEPT:

F. N H. Q J. T
G. P I. R

60. If T is on stand 2, which of the following sculptures must be selected?

F. M H. R J. U
G. N I. S

61. If P is not selected and R is on stand 1, which of the following lists, in alphabetical order,
those sculpture’s that must also be selected?
F. M,Q, T, and U
G. N, Q, S, and T
H. N, Q, S, and U
I. N, S, T, and U
J. Q, S, T, and U
62. If Q is displayed on a stand immediately adjacent to a stand on which R is displayed and
immediately adjacent to a stand on which S is displayed, which of the following must be
true?
F. N is on either stand 4 or stand 5
G. Q is on either stand 2 or stand 4
H. R is on either stand 1 or stand 3
I. S is on either stand 3 or stand 5
J. U is on either stand 2 or stand 4

Question 63-66: Seven meetings --- J, K, L, M, N, O and P ---- are to be schedules, one on each
day of a week that begins on Sunday.
 Meeting J must take place on Sunday.
 Meeting K must take place after both meeting L and Meeting M.
 Meeting N, O and P must take place on three consecutive days, not necessarily in that
order.
63. Which is the latest day of the week on which meeting L can take place?

F. Tuesday H. Thursday J. Saturday


G. Wednesday I. Friday

64. Which of the following must be true about the order of meeting?
F. L takes place after J
G. L takes place after O
H. N takes place after O
I. P takes place after P
J. takes place after P
65. If meeting O is on Saturday, then meeting K must take place on:

F. Monday H. Wednesday J. Friday


G. Tuesday I. Thursday

66. Which of the following represents a possible ordering of meetings on three consecutive
days?

F. JMK
G. KLO
H. MNJ
I. OJN
J. POM
Mental Ability Solved Paper

1. Mohan is 18th from either end of a row of boys? How many boys are there in that
row?
(A) 26 (B) 32 (C) 24 (D) 35

2. ‘Soldier’ is related to ‘Army’ in the same way as ‘Pupil’ is related to –


(A) Education (B) Teacher (C) Student (D) Class

3. ‘Kilogram’ is related to ‘Quintal’ in the same way as ‘Paisa’ is related to –


(A) Coin (B) Money (C) Cheque (D) Rupee

4. ‘Stammering’ is to ‘Speech’ as Deafness is to –


(A) Ear (B) Hearing (C) Noise (D) Commotion

5. ‘Guilt’ is to ‘Past’ as ‘Hope’ is to –


(A) Present (B) Future (C) Today (D) Hopeless
6. If the sequence of the alphabets is reversed which of the following would be the 14th
letter from your left ?
(A) N (B) L (C) O (D) None of these

7. Which letter is the 8th letter to the right of the letter, which is 12th from the left ?
(A) V (B) T (C) W (D) Y

8. Which letter is the 8th letter to the right of the letter which is 10th to the left of the last
but one letter from the right ?
(A) V (B) X (C) W (D) I

9. Typist : Typewriter : : Writer : ?


(A) Script (B) Pen (C) Paper (D) Book

10. Paint : Artist : : Wood : ?


(A) Furniture (B) Forest (C) Fire (D) Carpenter

11. acme : mace : : alga : ?


(A) glaa (B) gaal (C) laga (D) gala

12. EIGHTY : GIEYTH : : OUTPUT : ?


(A) UTOPTU (B) UOTUPT (C) TUOUTP (D) TUOTUP

13. ‘Medicine’ is related to ‘Patient’ in the same way as ‘Education’ is related to—
(A) Teacher (B) School (C) Student (D) Tuition
14. Fill in the missing letter in the following series—
S, V, Y, B, ?
(A) C (B) D (C) E (D) G
15. Select the correct option in place of the question mark.
AOP, CQR, EST, GUV, ?
(A) IYZ (B) HWX (C) IWX (D) JWX

Social ability

(Moral ethics)

You might also like